You are on page 1of 53

MANAGEMENT ADVISORY SERVICES

RISKS

INVESTMENT RISK
1
. Catherine & Co. has extra cash at the end of the year and is analyzing the best way to invest
the funds. The company should invest in a project only if
A. The expected return on the project exceeds the return on investments of comparable risk.
B. The return on investments of comparable risk exceeds the expected return on the project.
C. The expected return on the project is equal to the return on investments of comparable
risk.
Gleim
D. The return on investments of comparable risk equals the expected return on the project.
Interest-rate Risk
2
. Which of the following are components of interest-rate risk?
Gleim
A. Purchasing-power risk and default risk. C. Portfolio risk and reinvestment-rate risk.
B. Price risk and market risk.
D. Price risk and reinvestment-rate risk.
4. Long-term government bonds have:
A. Interest rate risk
B. Default risk

C. Market risk
D. None of the above

26. The portion of the risk that can be eliminated by diversification is called:
A. Unique risk
C. Interest rate risk
B. Market risk
D. Default risk

B&M

Market Risk
4
. The type of risk that is not diversifiable and even affects the value of a portfolio is (E)
A. Purchasing-power risk.
C. Nonmarket risk.
B. Market risk.
D. Interest-rate risk.

Gleim

C. Firm specific risk


D. All of the above

RPCPA, AICPA. CMA & CIA EXAMINATION QUESTIONS

B&M

In capital market analysis, the nonsystematic risk


a. Is correlated with qualitative aspects of the underlying entity.
b. Is correlated with quantitative aspects of the underlying entity.
c. Cannot easily be overcome by individual investors.
d. Is considered random.

B&M

Gleim

Which of the following statements is correct? (E)


a. Well diversified stockholders do not consider corporate risk when determining required
rates of return.
b. Undiversified stockholders, including the owners of small businesses, are more concerned
about corporate risk than market risk.
c. Empirical studies of the determinants of required rates of return (k) have found that only
market risk affects stock prices.
d. Market risk is important but does not have a direct effect on stock price because it only
affects beta.
e. All of the statements above are correct.
Brigham

In theory, the decision maker should view market risk as being of primary importance.
However, within-firm, or corporate, risk is relevant to a firms(M)
a. Well-diversified stockholders, because it may affect debt capacity and operating income.
b. Management, because it affects job stability.
c. Creditors, because it affects the firms credit worthiness.
d. Statements a and c are correct.
e. All of the statements above are correct.
Brigham

B&M

Purchasing-power Risk
Default Risk
3
. The marketable securities with the least amount of default risk are (E)
a. Federal government agency securities. c. Repurchase agreements.
b. U.S. Treasury securities.
d. Commercial paper.
CMA 0691 1-11

Nonmarket Risk or Company-specific Risk


27. The unique risk is also called the:
A. Unsystematic risk
B. Diversifiable risk

29. As the number of stocks in a portfolio is increased:


A. Unique risk decreases and approaches to zero
B. Market risk decrease
C. Unique risk decreases and becomes equal to market risk

Portfolio Risk
Total Risk
8
. The risk of a single asset is
A. Nonmarket risk.
B. Total risk.
9

An asset with high risk will have a(n)


A. Low expected return.

C. Portfolio risk.
D. Market risk.

Gleim

C. Increasing expected return.


Page 1 of 53

MANAGEMENT ADVISORY SERVICES


B. Lower price than an asset with low risk.

RISKS
D. High standard deviation of returns. Gleim

b.
c.
d.
e.

10

. Risk to a company is affected by both project variability and how project returns correlate with
those of the companys prevailing business. Overall company risk will be lowest when a
projects returns exhibit
CIA 1186 IV-39
a. Low variability and negative correlation. c. High variability and positive correlation.
b. Low variability and positive correlation.
d. High variability and no correlation.

Liquidity Risk
11
. The risk that securities cannot be sold at a reasonable price on short notice is called
A. Default risk.
C. Purchasing-power risk.
B. Interest-rate risk.
D. Liquidity risk.
CIA 1190 IV-51
12

. When purchasing temporary investments, which one of the following best describes the risk
associated with the ability to sell the investment in a short period of time without significant
price concessions? (E)
A. Interest rate risk.
C. Financial risk.
B. Purchasing power risk.
D. Liquidity risk.
CMA 0697 1-11

Business Risk
13
. Business risk is the risk inherent in a firm's operations that excludes financial risk. It depends
on all of the following factors except (E)
A. Amount of financial leverage.
C. Demand variability.
B. Sales price variability.
D. Input price variability.
Gleim
14

. Business risk excludes such factors as


A. Financial risk.
B. Amount of operating leverage.

C. Demand variability.
D. Fluctuations in suppliers' prices.

Gleim

1. A decrease in the debt ratio will generally have no effect on __________ ______. (E)
a. Financial risk.
b. Total risk.
c. Business risk.
d. Market risk.
Brigham
2. Business risk is concerned with the operations of the firm. Which of the following is not
associated with (or not a part of) business risk? (E)
a. Demand variability.
RPCPA, AICPA. CMA & CIA EXAMINATION QUESTIONS

Sales price variability.


The extent to which operating costs are fixed.
Changes in required returns due to financing decisions.
The ability to change prices as costs change.

Brigham

15

. Which of the following affects a firms business risk? (E)


a. The level of uncertainty about future sales.
b. The degree of operating leverage.
c. The degree of financial leverage.
d. Statements a and b are correct.
e. All of the statements above are correct.

Financial Risk
*. Which of the following would increase risk? (M)
a. Increase the level of working capital.
b. Change the composition of working capital to include more liquid assets.
c. Increase the amount of short-term borrowing.
d. Increase the amount of equity financing.

Brigham

RPCPA 1091

16

. A firms financial risk is a function of how it manages and maintains its debt. Which one of the
following sets of ratios characterizes the firm with the greatest amount of financial risk?
A. High debt-to-equity ratio, high interest coverage ratio, stable return on equity.
B. Low debt-to-equity ratio, low interest coverage ratio, volatile return on equity.
C. High debt-to-equity ratio, low interest coverage ratio, volatile return on equity.
D. Low debt-to-equity ratio, high interest coverage ratio, stable return on equity. CMA 1291 14

Business and financial risk


3. Which of the following statements is most correct? (E)
a. A firms business risk is solely determined by the financial characteristics of its industry.
b. The factors that affect a firms business risk are determined partly by industry
characteristics and partly by economic conditions. Unfortunately, these and other factors
that affect a firms business risk are not subject to any degree of managerial control.
c. One of the benefits to a firm of being at or near its target capital structure is that financial
flexibility becomes much less important.
d. The firms financial risk may have both market risk and diversifiable risk components.
Brigham
Page 2 of 53

MANAGEMENT ADVISORY SERVICES


Exchange-rate Risk
17
. The risk of loss because of fluctuations in the relative value of foreign currencies is called
A. Expropriation risk.
C. Multinational beta.
B. Sovereign risk.
D. Exchange rate risk.
CIA 1191 IV-60

RISKS
18

. O & B Company, a U.S. corporation, is in possession of accounts receivable denominated in


German deutsche marks. To what type of risk are they exposed? (E)
A. Liquidity risk.
C. Exchange-rate risk.
B. Business risk.
D. Price risk.
Gleim

19

. Bonner Electronics has subsidiaries in several international locations and is concerned about
its exposure to foreign exchange risk. In countries where currency values are likely to fall,
Bonner should encourage all of the following policies except
A. Granting trade credit whenever possible.
B. Investing excess cash in inventory or other real assets.
C. Purchasing materials and supplies on a trade credit basis.
CFM Sample Q. 5
D. Borrowing local currency funds if an appropriate interest rate can be obtained.

20

. A firm may seek to avoid exchange-rate risk by


A. Maintaining a net monetary debtor position in countries with strengthening currencies.
B. Maintaining a net monetary creditor position in countries with weakening currencies.
C. Avoiding diversification of foreign-currency transactions.
Gleim
D. Buying forward exchange contracts to cover liabilities denominated in a foreign currency.

Cultural Risk
56. A U.S. manufacturer of which of the following goods would be likely to face the most cultural
risks in operating globally?
a. Furniture
c. Clothing
b. Automobiles
d. Food
Barfields
57. A U.S. manufacturer of which of the following goods would be likely to face the fewest cultural
risks in operating globally?
a. Toys
c. Clothing
b. Food
d. Furniture
Barfields
Political Risk
58. Which of the following would be considered a political risk in doing business globally?
a. Asset expropriation
c. Workplace diversity
b. Inflation
d. All of the above
Barfields
21

. Political risk may be reduced by


A. Entering into a joint venture with another foreign company.

RPCPA, AICPA. CMA & CIA EXAMINATION QUESTIONS

Page 3 of 53

MANAGEMENT ADVISORY SERVICES


B. Making foreign operations dependent on the domestic parent for technology, markets, and
supplies.
C. Refusing to pay higher wages and higher taxes.
D. Financing with capital from a foreign country.
Gleim
Comprehensive
*. All of the following statements are correct except:
a. The matching of asset and liability maturities is considered desirable because this strategy
minimizes interest rate risk.
b. Default risk refers to the inability of the firm to pay off its maturing obligations.
c. The matching of assets and liability maturities lowers default risk.
d. An increase in the payables deferral period will lead to a reduction in the need to nonspontaneous funding.
RPCPA 1095

RISKS
RISK MANAGEMENT METHODS
Portfolio Theory
1. Portfolio Theory was first developed by:
A. Merton Miller
B. Franco Modigliani

C. Harry Markowitz
D. Richard Breadey

B&M

12. A portfolio will a usually contain:


A. One riskless asset
B. Two or more assets

C. One risky asset


D. None of the above

B&M

Portfolio Management
Efficient Portfolio
25. Efficient portfolios are those which offer:
A. Highest expected return for a given level of risk
B. Highest risk for a given level of expected return
C. The maximum risk and expected return
D. All of the above

B&M

33. Efficient portfolios are portfolios that:


A. Offer the highest rate of return for a given level of risk
B. Offer the lowest rate of return for a given level of risk
C. Offer the lowest level of risk for a given rate of return
D. A and C

B&M

Feasible Portfolio
22
. A feasible portfolio that offers the highest expected return for a given risk or the least risk for a
given expected return is a(n)
A. Optimal portfolio.
C. Efficient portfolio.
B. Desirable portfolio.
D. Effective portfolio.
Gleim
Optimal Portfolio
23
. An optimal portfolio of investments is (E)
A. Efficient because it offers the highest expected return.
B. Any portfolio chosen from the efficient set of portfolios.
C. Any portfolio chosen from the feasible set of portfolios.
D. Tangent to the investor's highest indifference curve.
RPCPA, AICPA. CMA & CIA EXAMINATION QUESTIONS

Gleim
Page 4 of 53

MANAGEMENT ADVISORY SERVICES

RISKS

24

. A company uses portfolio theory to develop its investment portfolio. If the company wishes to
obtain optimal risk reduction through the portfolio effect, it should make its next investment in
A. An investment that correlates negatively to the current portfolio holdings.
B. An investment that is uncorrelated to the current portfolio holdings.
C. An investment that is highly correlated to the current portfolio holdings.
D. An investment that is perfectly correlated to the current portfolio holdings. CIA 0591 IV-48

Minimum Variance Portfolio


24. Florida Company (FC) and Minnesota Company (MC) are both service companies. Their
historical return for the past three years are: FC: -10%,15%, 25%; MC: 10%, 6%, 32%. Which
portfolio is better?
A. Portfolio with 50% in FC and 50% in MC C. Investment in FC
B. Minimum variance portfolio
D. None of the above
B&M
26. Is the minimum variance portfolio an efficient portfolio?
A. Yes
C. Not necessarily
B. No

B&M

Well-Diversified Portfolio
30. In a well diversified portfolio, the type of risk remaining is:
A. Individual security risk
C. Total risk
B. Zero risk
D. Market risk

B&M

31. A well-diversified portfolio has negligible:


A. Systematic risk
B. Unique risk

C. Market risk
D. None of the above

B&M

Portfolio Matrix Analysis


25
. Which one of the following planning techniques is most likely to be used to determine which
business units will receive additional capital and which will be divested? (D)
A. Competitive strategies model.
C. Scenario development.
B. Portfolio matrix analysis.
D. Situational analysis.
CMA Samp Q3-9
Unsystematic Risk
26
In a well diversified portfolio (M)
a. market risk is negligible.
b. systematic risk is negligible.

Bodie
c. unsystematic risk is negligible.
d. nondiversifiable risk is negligible.

RPCPA, AICPA. CMA & CIA EXAMINATION QUESTIONS

Systematic Risk
27
. Capital Asset Pricing Theory asserts that portfolio returns are best explained by: (E)
a. economic factors.
c. systematic risk.
b. specific risk.
d. diversification.

Bodie

Standard Deviation vs. Beta Coefficient


23. Standard deviation and beta both measure risk, but they are different in that (E)
a. beta measures both systematic and unsystematic risk.
b. beta measures only systematic risk while standard deviation is a measure of total isk.
c. beta measures only unsystematic risk while standard deviation is a easure of total risk.
d. beta measures both systematic and unsystematic risk while standard deviation measures
only systematic risk.
Bodie
e. beta measures total risk while standard deviation measures only nonsystematic risk.
Market Price of Risk
28
. The market price of risk (M)
a. is the risk premium divided by the standard deviation of the market returns.
b. has a reward-to-risk ratio of [E(rM ) - rf]/2M.
c. is the price of a U. S. T-bill.
d. a and b.
e. a and c.

Bodie

Risk Level of Securities


29
. Which of the following classes of securities are listed in order from lowest risk/opportunity for
return to highest risk/opportunity for return? (E)
A. U.S. Treasury bonds; corporate first mortgage bonds; corporate income bonds; preferred
stock.
B. Corporate income bonds; corporate mortgage bonds; convertible preferred stock;
subordinated debentures.
C. Common stock; corporate first mortgage bonds; corporate second mortgage bonds;
corporate income bonds.
CIA 0589 IV-49
D. Preferred stock; common stock; corporate mortgage bonds; corporate debentures.
1. Which of the following portfolios have the least risk?
A. A portfolio of treasury bills
B. A portfolio of long term United States Government bonds
C. Standard and Poor's composite index
D. Portfolio of common stocks of small firms

B&M
Page 5 of 53

MANAGEMENT ADVISORY SERVICES

RISKS
B. Pure play risk (beta).
C. The weighted average of project risk (betas).
D. Accounting risk (beta).

30

. From the viewpoint of the investor, which of the following securities provides the least risk?
a. Mortgage bond.
c. Income bond.
b. Subordinated debenture.
d. Debentures.
CIA 1191 IV-50

CIA 0592 IV-49

36
31

. The expected rate of return for the stock of Corn Enterprises is 20%, with a standard deviation
of 15%. The expected rate of return for the stock of Must Associates is 10%, with a standard
deviation of 9%. The riskier stock is
A. Corn because its return is higher.
B. Corn because its standard deviation is higher.
C. Must because its standard deviation is higher.
D. Must because its coefficient of variation is higher.
CMA 0692 1-6

Risky Investment vs. Riskless Investment


32
. The difference between the required rate of return on a given risky investment and that on a
riskless investment with the same expected return is the
A. Risk premium.
C. Standard deviation.
B. Coefficient of variation.
D. Beta coefficient.
CIA 1192 IV-48
Capital Asset Pricing Model
33
. The capital asset pricing model deals with risk and rates of return of a
A. Single security.
B. Group of securities in a portfolio which follows a buy and hold strategy.
C. Portfolio and how a new security affects that portfolio.
D. Single fixed asset.
34

CIA 0585 IV-28

According to the Capital Asset Pricing Model (CAPM) a well diversified portfolio's rate of return
is a function of (E)
a. market risk
c. unique risk.
b. unsystematic risk
d. reinvestment risk.
Bodie

Beta Coefficient
37. The "beta" is a measure of:
A. Unique risk
B. Market risk

C. Total risk
D. None of the above

35

. The level of risk that concerns investors who supply capital to a diversified company is
A. Project risk (beta).

RPCPA, AICPA. CMA & CIA EXAMINATION QUESTIONS

B&M

. A company's beta value has decreased because of a change in its marketing strategy.
Consequently, the discount rate applied to expected cash flows of potential projects will be
A. Reduced.
C. Unchanged.
B. Increased.
D. Zero.
CIA 1185 IV-26

Portfolio Beta
39. The beta of market portfolio is:
A. 0
B. +0.5

C. +1.0
D. 1.0

B&M

40. The beta of a risk-free portfolio is:


A. 0
B. +0.5

C. +1.0
D. 1.0

B&M

34. Beta of Treasury bills portfolio is:


A. Zero
B. +0.5

C. 1.0
D. +1.0

B&M

44. A stock with a beta of zero would be expected to:


A. Have a rate of return equal to the risk-free rate
B. Have a rate of return equal to the market risk premium
C. Have a rate of return equal to zero
D. Have a rate of return equal to the market rate of return

B&M

Equity Beta
37. If beta of debt is zero, then the beta of equity is equal to:
A. (1 + Debt-equity ratio)(beta of assets)
C. (Beta of assets)/(debt-equity ratio)
B. (Debt-equity ratio)(beta of assets)
D. None of the above
B&M
37. In many situations debt beta can be safely assumed to be zero. Under this assumption, equity
beta can be expressed as: [E = market value of equity and D = market value of debt]
A. equity beta = (1-(D/E.) (asset beta)
C. equity beta = (asset beta)/(1+(D/E.)
B. equity beta = (1+(D/E.) (asset beta)
D. None of the above
B&M
Page 6 of 53

MANAGEMENT ADVISORY SERVICES

RISKS

Asset Beta, Debt Beta & Equity Beta


32. Which of the following is true?
A. bD > bA > bE
B. bE > bA > bD

C. bA > bE > bD
D. None of the above are true

34. Which of the following is true?


A. bD < bA < bE
B. bE < bA < bD

C. bA < bE < bD
D. None of the above are true

Standard Deviation
38. The variance or standard deviation is a measure of:
A. Total risk
C. Market risk
B. Unique risk
D. None of the above

B&M

B&M

28. In the formula for calculating the variance of an N-asset portfolio, how many are variance
terms?
A. [N(N-1)]/2
C. N
B. N^2
D. None of the above
B&M

B&M

32. The variance formula for a four stock portfolio contains:


A. 4 individual variance terms and 6 unique covariance terms
B. 3 individual variance terms and 6 unique covariance terms
C. 6 individual variance terms and 9 unique covariance terms
D. 6 individual variance terms and 6 unique covariance terms

2. Investments A and B both offer an expected rate of return of 12%. If the standard deviation of
A is 20% and that of B is 30%, then investors would:
A. Prefer A to B
B. Prefer B to A
C. Prefer a portfolio of A and B
D. Cannot answer without knowing investor's risk preferences
B&M
Coefficient of Variation
24. Which of the following can be computed and compared for each alternative to determine the
relative riskiness of investments that have different levels of expected return?
A. coefficient of variation
C. standard deviation
B. variance
D. expected value
Carter & Usry
37

. The expected rate of return for the stock of Cornhusker Enterprises is 20%, with a standard
deviation of 15%. The expected rate of return for the stock of Mustang Associates is 10% with
a standard deviation of 9%. The riskier stock is (M)
a. Cornhusker because the return is higher.
b. Cornhusker because the standard deviation is higher.
c. Mustang because the standard deviation is higher.
d. Mustang because the return is lower.
e. Mustang because the coefficient of variation is higher.
CMA 0692 1-6

RPCPA, AICPA. CMA & CIA EXAMINATION QUESTIONS

Variance
27. In the formula for calculating the variance of an asset portfolio, how many are covariance
terms?
A. [N(N-1)]/2
C. N
B. N^2
D. None of the above
B&M

B&M

Covariance
38
. Which of the following specifically measures the volatility of returns together with their
correlation with the returns of other securities? (M)
A. Variance.
C. Coefficient of variation.
B. Standard deviation.
D. Covariance.
Gleim
39

. If the covariance of stock A with stock B is -.0076, then what is the covariance of stock B with
stock A?
A. +.0076
C. Greater than .0076.
B. -.0076
D. Less than -.0076.
Gleim

Indifference Curve
40
. An indifference curve represents combinations of portfolios having equal utility to the investor.
Given that risk and returns are plotted on the horizontal and vertical axes, respectively, and
that the investor is risk averse, the curve has
A. An increasingly steeper slope if the investor is less risk averse.
B. A decreasingly negative slope if the investor's utility increases.
C. An increasingly positive slope.
D. A decreasingly positive slope.
Gleim

Page 7 of 53

MANAGEMENT ADVISORY SERVICES


Expected Rate of Return
3. When stocks with the same expected return are combined into a portfolio, the expected return
of the portfolio is:
A. Less than the average expected return value of the stocks
B. Greater than the average expected return of the stocks
C. Equal to the average expected return of the stocks
D. Impossible to predict
B&M
41

. An investor uses the capital asset pricing model (CAPM) to evaluate the risk-return
relationship on a portfolio of stocks held as an investment. Which of the following would not be
used to estimate the portfolio's expected rate of return? (D)
A. Expected risk premium on the portfolio of stocks.
B. Interest rate for the safest possible investment.
C. Expected rate of return on the market portfolio.
D. Standard deviation of the market returns.
CIA 1193 IV-47

RISKS
A. +1
B. 0

C. 0.5
D. 1

B&M

11. Maximum diversification is obtained by combining two stocks with a correlation coefficient
equal to:
A. +1.0
C. 1.0
B. 0.0
D. +0.5
B&M
Correlation
9. The correlation measures the:
A. Rate of movements of the return of individual stocks
B. Direction of movement of the return of individual stocks
C. Direction of movement between the returns of two stocks
D. Stock market volatility

B&M

42

Expected Return Beta Relationship


25. The expected return-beta relationship (M)
a. is the most familiar expression of the CAPM to practitioners.
b. refers to the way in which the covariance between the returns on a stock and returns on the
market measures the contribution of the stock to the variance of the market portfolio, which is
beta.
c. assumes that investors hold well-diversified portfolios
d. all of the above are true.
Bodie
Comprehensive
12 According to the Capital Asset Pricing Model (CAPM), which one of the following statements is
false? (M)
a. The expected rate of return on a security decreases in direct proportion to a decrease n the
risk-free rate.
b. The expected rate of return on a security increases directly with its beta.
c. A fairly priced security has an alpha of zero.
d. In equilibrium, all securities lie on the security market line.
Bodie
Two-Stocks Portfolio
Correlation Coefficient
36. For a two-stock portfolio, the maximum reduction in risk occurs when the correlation coefficient
between the two stocks is:
RPCPA, AICPA. CMA & CIA EXAMINATION QUESTIONS

. The returns on two stocks can be correlated in values except those that are
A. Positive.
C. Neutral.
B. Negative.
D. Skewed.

Gleim

Variance
29. If the correlation between two stocks is +1, then a portfolio combining these two stocks will
have variance that is:
A. Less than the weighted average of the two individual variances
B. More than the weighted average of the two individual variances
C. Equal to the weighted average of the two individual variances
D. None of the above
B&M
Expected Return
4. Stock A has an expected return of 20%, and stock B has an expected return of 12%. The risk
of Stock A as measured by the variance of the returns is twice that of stock B. If the two stocks
are combined equally in a portfolio, what would be the expected return of the portfolio?
A. 16%
C. 20%
B. 12%
D. Need more information to answer B & M
Hedging
43
. When a firm finances each asset with a financial instrument of the same approximate maturity
as the life of the asset, it is applying
Page 8 of 53

MANAGEMENT ADVISORY SERVICES


A. Working capital management.
B. Return maximization.

RISKS
C. Financial leverage.
D. A hedging approach.

C. A commitment today to purchase a product on a specific future date at a price determined


today.
D. A commitment today to purchase a product only when its price increases above its current
exercise price.
Gleim

CMA 1291 1-13

44

. Contracts to hedge risk by exchanging cash flows include (E)


Gleim
A.
B.
Interest-Rate Swaps
Yes
Yes
Currency Swaps
Yes
No

C.
No
Yes

D.
No
No

45

. The use of derivatives to either hedge or speculate results in


A. Increased risk regardless of motive.
B. Decreased risk regardless of motive.
C. Offset risk when hedging and increased risk when speculating.
D. Offset risk when speculating and increased risk when hedging.

Gleim

46

. A company has recently purchased some stock of a competitor as part of a long-term plan to
acquire the competitor. However, it is somewhat concerned that the market price of this stock
could decrease over the short run. The company could hedge against the possible decline in
the stock's market price by
CIA 0590 IV-57
A. Purchasing a call option on that stock.
C. Selling a put option on that stock.
B. Purchasing a put option on that stock.
D. Obtaining a warrant option on that stock.

Duration Hedging
47
. Duration hedging involves hedging interest-rate risk by matching the duration of assets with
the duration of liabilities. Which of the following is a true statement about duration hedging?
(M)
A. If duration increases, the volatility of the price of a debt instrument decreases.
B. The goal of duration hedging is to equate the duration of assets with the duration of
liabilities.
C. The firm is immunized against interest-rate risk when the total price change for assets
equals the total price change for liabilities.
D. Duration is higher if the nominal rate on a debt instrument is higher.
Gleim
Forward Contract
48
. A forward contract involves (E)
A. A commitment today to purchase a product on a specific future date at a price to be
determined some time in the future.
B. A commitment today to purchase a product some time during the current day at its present
price.
RPCPA, AICPA. CMA & CIA EXAMINATION QUESTIONS

49

. If a corporation holds a forward contract for the delivery of U.S. Treasury bonds in 6 months
and, during those 6 months, interest rates decline, at the end of the 6 months the value of the
forward contract will have
A. Decreased.
B. Increased.
C. Remained constant.
D. Any of the answers may be correct, depending on the extent of the decline in interest
rates.
Gleim

Futures Contract
50
. A distinguishing feature of a futures contract is that
A. Performance is delayed.
C. Delivery is to be on a specific day. Gleim
B. It is a hedge, not a speculation.
D. The price is marked to market each day.
51

. An automobile company that uses the futures market to set the price of steel to protect a profit
against price increases is an example of
A. A short hedge.
B. A long hedge.
C. Selling futures to protect the company from loss.
D. Selling futures to protect against price declines.
Gleim

Interest rate futures contract


Interest rate Swap
52
. In an interest rate swap, the first company
A. Sells its right to low interest rate financing at a financial institution to the second company
that is seeking to borrow funds.
B. Agrees to service the debt of the second company by making interest payments directly to
the bank of the second company, while the second company agrees in exchange to make
interest payments to the bank of the first company.
C. Buys the outstanding public debt of the second company and swaps the interest
payments it receives on that debt for the interest payments it must make on its own debt.

Page 9 of 53

MANAGEMENT ADVISORY SERVICES

RISKS

D. Agrees to exchange with the second company the difference between the interest charges
on its own borrowings and the interest charges on the borrowings of the second company.
CIA 0596 IV-29
Interest Rate & Currency Swap
53
. Contracts to hedge risk by exchanging cash flows include
Gleim
A.
B.
Interest-Rate Swaps
Yes
Yes
Currency Swaps
Yes
No

C.
No
Yes

D.
No
No

METHODS OF ANALYZING RISK


Sensitivity Analysis
54
. Which of the following approaches would best analyze the risk of increasing the price of a
table by $50? (E)
A. Sensitivity analysis.
C. Informal method.
B. Simulation analysis.
D. Certainty equivalent adjustments. Gleim
Simulation Analysis
55
. Which method for measuring risk considers both the sensitivity of changing NPVs and the
range of values of the variables that are changed?
A. Simulation analysis.
C. Sensitivity analysis.
B. The Capital Asset Pricing Model.
D. Certainty equivalent adjustments. Gleim
Analysis of Pricing Technique
50. The acronym APT stands for:
A. Arbitrage Pricing Model
B. Asset Pricing Tool

C. Analysis of Pricing Technique


D. Analysis Pricing Theory

51. A "factor" in APT is a variable that:


A. Affects the return of risky assets in a systematic manner
B. Correlates with risky asset returns in an unsystematic manner
C. Is purely "noise"
D. Affects the return of a risky asset in a random manner
Three-factor Model
55. The three factors in the Three-Factor Model are:
A. Market factor
C. Book-to-market factor
B. Size factor
D. All of the above
RPCPA, AICPA. CMA & CIA EXAMINATION QUESTIONS

B&M

B&M

RISK-ADJUSTED DISCOUNT RATE


56
. Mega Inc., a large conglomerate with operating divisions in many industries, uses risk-adjusted
discount rates in evaluating capital investment decisions. Consider the following statements
concerning Mega's use of risk-adjusted discount rates.
I. Mega may accept some investments with internal rates of return less than Mega's overall
average cost of capital.
II. Discount rates vary depending on the type of investment.
III. Mega may reject some investments with internal rates of return greater than the cost of
capital.
IV. Discount rates may vary depending on the division.
Which of the above statements are correct? (D)
A. I and III only.
C. II, III, and IV only.
B. II and IV only.
D. I, II, III, and IV.
CMA Samp Q4-5
57

. Dick Boe Enterprises, an all-equity firm, has a corporate beta coefficient of 1.5. The financial
manager is evaluating a project with an expected return of 21 percent, before any risk
adjustment. The risk-free rate is 10 percent, and the required rate of return on the market is 16
percent. The project being evaluated is riskier than Boes average project, in terms of both
beta risk and total risk. Which of the following statements is most correct? (E)
a. The project should be accepted since its expected return (before risk adjustment) is
greater than its required return.
b. The project should be rejected since its expected return (before risk adjustment) is less
than its required return.
c. The accept/reject decision depends on the risk-adjustment policy of the firm. If the firms
policy were to reduce a riskier-than-average projects expected return by 1 percentage
point, then the project should be accepted.
d.
Riskier-than-average projects should have their expected returns increased to
reflect their added riskiness. Clearly, this would make the project acceptable regardless of
the amount of the adjustment.
e. Projects should be evaluated on the basis of their total risk alone. Thus, there is insufficient information in the problem to make an accept/reject decision.
Brigham

58

B&M

. Assume you are the director of capital budgeting for an all-equity firm. The firms current cost
of equity is 16 percent; the risk-free rate is 10 percent; and the market risk premium is 5
percent. You are considering a new project that has 50 percent more beta risk than your firms
assets currently have, that is, its beta is 50 percent larger than the firms existing beta. The
Page 10 of 53

MANAGEMENT ADVISORY SERVICES

RISKS

expected return on the new project is 18 percent. Should the project be accepted if beta risk is
the appropriate risk measure? Choose the correct statement. (M)
a. Yes; its expected return is greater than the firms cost of capital.
b. Yes; the projects risk-adjusted required return is less than its expected return.
c. No; a 50 percent increase in beta risk gives a risk-adjusted required return of 24 percent.
d. No; the projects risk-adjusted required return is 2 percentage points above its expected
return.
e. No; the projects risk-adjusted required return is 1 percentage point above its expected
return.
Brigham
59

. Assume you are the director of capital budgeting for an all-equity firm. The firms current cost
of equity is 16 percent; the risk-free rate is 10 percent; and the market risk premium is 5
percent. You are considering a new project that has 50 percent more beta risk than your firms
assets currently have, that is, its beta is 50 percent larger than the firms existing beta. The
expected return on the new project is 18 percent. Should the project be accepted if beta risk is
the appropriate risk measure? Choose the correct statement. (M)
a. Yes; its expected return is greater than the firms cost of capital.
b. Yes; the projects risk-adjusted required return is less than its expected return.
c. No; a 50 percent increase in beta risk gives a risk-adjusted required return of 24 percent.
d. No; the projects risk-adjusted required return is 2 percentage points above its expected
return.
e. No; the projects risk-adjusted required return is 1 percentage point above its expected
return.
Brigham

COST OF CAPITAL
42. Cost of capital is (E)
a. The interest rate an entity must pay to borrow money.
b. The return an entitys stockholders expect on their investment .
c. The rate of return the entity can earn from investing available cash.
d. A concept of managerial finance incorporating all of the above.

L&H

60

. The theory underlying the cost of capital is primarily concerned with the cost of
A. Long-term funds and old funds.
B. Short-term funds and new funds.
C. Long-term funds and new funds.
D. Any combination of old or new, short-term or long-term funds.
CMA 0692 1-13

61

. Management knowledge of the cost of capital is useful for each of the following except (D)
a. Making capital investment decisions.
b. Managing working capital.
c. Setting the maximum rate of return on new investments.
d. Evaluating performance.
Gleim

62

. In referring to the graph of a firm's cost of capital, if e is the current position, which one of the
following statements best explains the saucer or U-shaped curve?

Cost of
Capital
(percent)
e

L&H

13. Cost of capital is


a. The amount the company must pay for its plant assets.
b. The dividends a company must pay on its equity securities.
c. The cost the company must incur to obtain its capital resources.
d. The cost the company is charged by investment bankers who handle the issuance of
equity or long-term debt securities.
L&H
RPCPA, AICPA. CMA & CIA EXAMINATION QUESTIONS

26. A dollar now is worth more than a dollar to be received in the future because of
a. Inflation.
c. The opportunity cost of waiting.
b. Uncertainty.
d. None of the above.

Debt-to-Equity Ratio
A. The composition of debt and equity does not affect the firm's cost of capital.
B. The cost of capital is almost always favorably influenced by increases in financial
leverage.
C. The financial markets will penalize firms that borrow even in moderate amounts.
D. Use of at least some debt financing will enhance the value of the firm.
CMA 1288 1-5
26. The pre-tax cost of capital is higher than the after-tax cost of capital because (E)
a. interest expense is deductible for tax purposes.
b. principal payments on debt are deductible for tax purposes.
c. the cost of capital is a deductible expense for tax purposes.
Page 11 of 53

MANAGEMENT ADVISORY SERVICES


d. dividend payments to stockholders are deductible for tax purposes.

RISKS
Barfield

a.
b.
c.
d.

63

. The overall cost of capital is the


A. Rate of return on assets that covers the costs associated with the funds employed.
B. Average rate of return a firm earns on its assets.
C. Minimum rate a firm must earn on high-risk projects.
D. Cost of the firm's equity capital at which the market value of the firm will remain
unchanged.
CMA 0692 1-11

*.

Which of these statements are pertinent to cost of capital? (D)


1. It is the expected return that investors demand for a given level of risk.
2. It may be employed as a benchmark for the evaluation of performance.
3. For investment decisions, it must be based on the current or prospective cost of the
various capital components rather than on their historical costs.
4. It may also be used in acquisition analysis, liquidation studies and source of financing
decisions.
5. It may differ from the hurdle rate used to reflect relative risk attributed to a specific project,
division, or business unit.
RPCPA 1094
a. All five statements.
c. Statements 1, 2, 3, and 4 only.
b. Statements 1, 2 and 3 only.
d. Statements 1, 2, 4 and 5 only.

The stated interest paid on a bank loan.


Assets that are considered obsolete that maintain a net book value.
Decelerated depreciation.
Lending funds to a supplier at a lower-than-market rate in exchange for receiving the
suppliers products at a discount.
CMA 0689 1-25

65

. The explicit cost of debt financing is the interest expense. The implicit cost(s) of debt financing
is (are) the (D)
a. Increase in the cost of debt as the debt-to-equity ratio increases.
b. Increases in the cost of debt and equity as the debt-to-equity ratio increases.
c. Increase in the cost of equity as the debt-to-equity ratio decreases.
CMA 1291 1-2
d. Decrease in the weighted-average cost of capital as the debt-to-equity ratio increases.

24. Which of the following is true?


a. Companies can raise common equity only by issuing new shares of common stock.
b. There is no opportunity cost associated with use of retained earnings as a source of
common equity.
c. Most large mature firms issue new shares of common stock on a regular basis.
d. Companies can raise common equity by issuing new shares of common stock and
through retained earnings.
S, S & S
25. Which of the following is incorrect?
a. The after-tax cost of debt for a firm with losses is equal to the interest rate on the debt.
b. Most debt is placed privately and thus there is no flotation cost.
c. Flotation costs for preferred stock are higher than for debt.
d. Firms always pay dividends on their common stock issues because of the ease with which
common shareholders can assume control of the firm.
S, S & S
Imputed Costs vs. Explicit Costs
64
. All of the following are examples of imputed costs except (M)
RPCPA, AICPA. CMA & CIA EXAMINATION QUESTIONS

Page 12 of 53

MANAGEMENT ADVISORY SERVICES

RISKS

Cost of Debt Capital


66
. Which of the following statements is most correct? (E)
a. Since the money is readily available, the cost of retained earnings is usually a lot cheaper
than the cost of debt financing.
b. When calculating the cost of preferred stock, a company needs to adjust for taxes,
because preferred stock dividends are tax deductible.
c. When calculating the cost of debt, a company needs to adjust for taxes, because interest
payments are tax deductible.
d. Statements a and b are correct.
e. Statements b and c are correct.
Brigham

II.
III.
IV.
V.
a.
b.

If the return on total assets is 10% and if the return on common stockholders equity is 12%
then (D)
a. The after-tax cost of long-term debt is probably greater than 10%.
b. The after-tax cost of long-term debt is 12%.
c. Leverage is negative.
d. The after-tax cost of long-term debt is probably less than 10%.
RPCPA 1095

Which of the following statements is most correct? (M)


a. Suppose a firm is losing money and thus, is not paying taxes, and that this situation is
expected to persist for a few years whether or not the firm uses debt financing. Then the
firms after-tax cost of debt will equal its before-tax cost of debt.
b. The component cost of preferred stock is expressed as k p(1 - T), because preferred stock
dividends are treated as fixed charges, similar to the treatment of debt interest.
c. The reason that a cost is assigned to retained earnings is because these funds are
already earning a return in the business; the reason does not involve the opportunity cost
principle.
Brigham
d. The bond-yield-plus-risk-premium approach to estimating a firms cost of common equity
involves adding a subjectively determined risk-premium to the market risk-free bond rate.

Gleim

68

. If k is the cost of debt and t is the marginal tax rate, the after-tax cost of debt k, is best
represented by the formula
a. ki = k/t
c. ki = k(t)
b. ki =k/(1 t)
d. ki = k (1 t)
CMA 1288 1-3

69

. If Brewer Corporation's bonds are currently yielding 8% in the marketplace, why is the firm's
cost of debt lower? (E)
A. Market interest rates have increased.
B. Additional debt can be issued more cheaply than the original debt.
C. There should be no difference; cost of debt is the same as the bonds' market yield.
D. Interest is deductible for tax purposes.
CMA 0692 1-12

70

The interest rate on the bonds is greater for the second alternative consisting of pure debt than
it is for the first alternative consisting of both debt and equity because
A. The diversity of the combination alternative creates greater risk for the investor.
B. The pure debt alternative would flood the market and be more difficult to sell.
C. The pure debt alternative carries the risk of increasing the probability of default.
D. The combination alternative carries the risk of increasing dividend payments.

71

If a $1,000 bond sells for $1,125, which of the following statements are correct?
I. The market rate of interest is greater than the coupon rate on the bond.

CMA 0695 1-6

*.

67

. In computing the cost of capital, the cost of debt capital is determined by (E)
a. Annual interest payment divided by the proceeds from debt issuance.
b. Interest rate times (1 the firms tax rate)
c. Annual interest payment divided by the book value of the debt.
d. The capital asset pricing model.

The coupon rate on the bond is greater than the market rate of interest.
The coupon rate and the market rate are equal.
The bond sells at a premium.
The bond sells at a discount.
I and IV.
c. II and IV.
I and V.
d. II and V.

Marginal Cost of Debt


72
. The marginal cost of debt for a firm is defined as the interest rate on <List A> debt minus the
<List B>. (M)
CIA 0594 IV-48
List A
List B
A.
New
Firm's marginal tax rate
B.
Outstanding
Firm's marginal tax rate
C.
New
Interest rate times the firm's marginal tax rate
D.
Outstanding
Interest rate times the firm's marginal tax rate
73

RPCPA, AICPA. CMA & CIA EXAMINATION QUESTIONS

. Which of the following statements is most correct? (E)


Page 13 of 53

MANAGEMENT ADVISORY SERVICES


a. If a companys tax rate increases but the yield to maturity of its noncallable bonds remains
the same, the companys marginal cost of debt capital used to calculate its weighted
average cost of capital will fall.
b. All else equal, an increase in a companys stock price will increase the marginal cost of
retained earnings, ks.
c. All else equal, an increase in a companys stock price will increase the marginal cost of
issuing new common equity, ke.
d. Statements a and b are correct.
e. Statements b and c are correct.
Brigham
Cost of Debt & Cost of Preferred Stock
17. The basis for measuring the cost of capital derived from bonds and preferred stock,
respectively, is the (M)
A. after-tax rate of interest for bonds and stated annual dividend rate for preferred stock
B. pretax rate of interest for bonds and stated annual dividend rate less the expected
earnings per share for preferred stock
C. pretax rate of interest for bonds and stated annual dividend rate for preferred stock
D. after-tax rate of interest for bonds and stated annual dividend rate less the expected
earnings per share for preferred stock
AICPA adapted
Cost of Debt vs. Cost of Equity Capital
74
. In general, it is more expensive for a company to finance with equity capital than with debt
capital because (E)
A. Long-term bonds have a maturity date and must therefore be repaid in the future.
B. Investors are exposed to greater risk with equity capital.
C. Equity capital is in greater demand than debt capital.
D. Dividends fluctuate to a greater extent than interest rates.
CMA 0690 1-15
Cost of Equity Capital
Cost of Preferred Stock
75
. Which of the following statements is most correct? (M)
a. The before-tax cost of preferred stock may be lower than the before-tax cost of debt, even
though preferred stock is riskier than debt.
b. If a companys stock price increases, this increases its cost of common equity.
c. If the cost of equity capital is low enough, it may be cheaper to issue common stock than it
is to finance projects with retained earnings.
d. Statements a and b are correct.
Brigham
RPCPA, AICPA. CMA & CIA EXAMINATION QUESTIONS

RISKS
Cost of Common Equity
. Which of the following factors in the discounted cash flow (DCF) approach to estimating the
cost of common equity is the least difficult to estimate? (E)
a. Expected growth rate, g.
c. Required return, ks.
b. Dividend yield, D1/P0.
d. Expected rate of return, ks .
Brigham
76

. Assume that nominal interest rates just increased substantially but that the expected future
dividends for a company over the long run were not affected. As a result of the increase in
nominal interest rates, the company's stock price should
A. Increase.
C. Stay constant.
CIA 0593 IV-49
B. Decrease.
D. Change, but in no obvious direction.

77

. The market value of a firms outstanding common shares will be higher, everything else equal,
if (M)
a. Investors have a lower required return on equity.
b. Investors expect lower dividend growth.
c. Investors have longer expected holding periods.
d. Investors have shorter expected holding periods.
CIA 1196 IV-25

Cost of Retained Earnings


78
. When calculating the cost of capital, the cost assigned to retained earnings should be (E)
A. Zero.
B. Lower than the cost of external common equity.
C. Equal to the cost of external common equity.
D. Higher than the cost of external common equity.
CIA 1195 IV-43
79

. Which of the following statements is most correct? (M)


a. The cost of retained earnings is the rate of return stockholders require on a firms common
stock.
b. The component cost of preferred stock is expressed as k p(1 - T), because preferred stock
dividends are treated as fixed charges, similar to the treatment of debt interest.
c. The bond-yield-plus-risk-premium approach to estimating a firms cost of common equity
involves adding a subjectively determined risk-premium to the market risk-free bond rate.
d. The higher the firms flotation cost for new common stock, the more likely the firm is to use
preferred stock, which has no flotation cost.
Brigham

Marginal Cost of Capital


80
. If a company has a higher dividend-payout ratio, then, if all else if equal, it will have
Page 14 of 53

MANAGEMENT ADVISORY SERVICES


a.
b.
c.
d.

A higher marginal cost of capital.


A lower marginal cost of capital.
A higher investment opportunity schedule.
A lower investment opportunity schedule.

RISKS

CIA 0597 IV-53

81

. The firms marginal cost of capital (E)


a. Should be the same as the firms rate of return on equity.
b. Is unaffected by the firms capital structure.
CMA 1291 1-8
c. In inversely related to the firms required rate of return used in capital budgeting.
d. Is a weighted-average of the investors required returns on debt and equity.

Dividend Growth Model


82
. Which of the following criteria theoretically should be used to determine the valuation of
common stock? (E)
A. Book value.
C. Beta coefficient.
B. Dividends.
D. Standard deviation of returns.
Gleim
83

. Which of the following is directly applied in determining the value of a stock when using the
dividend growth model?
A. The firm's capital structure.
B. The firm's cash flows.
C. The firm's liquidity.
D. The investor's required rate of return on the firm's stock.
CIA 1190 IV-53

84

. The three elements needed to estimate the cost of equity capital for use in determining a firm's
weighted-average cost of capital are (E)
A. Current dividends per share, expected growth rate in dividends per share, and current
book value per share of common stock.
B. Current earnings per share, expected growth rate in dividends per share, and current
market price per share of common stock.
C. Current earnings pers share, expected growth rate in earnings per share, and current
book value per share of common stock.
D. Current dividends per share, expected growth rate in dividends per share, and current
market price per share of common stock.
CMA 1291 1-3

25. The value of the stock:


A. Increases as the dividend growth rate increases
B. Increases as the required rate of return decreases
RPCPA, AICPA. CMA & CIA EXAMINATION QUESTIONS

C. Increases as the required rate of return increases


D. Both A and B

B&M

Dividend Growth Rate


14. Dividend growth rate for a stable firm can be estimated as:
A. Plow back rate * the return on equity (ROE)
B. Plow back rate / the return on equity (ROE)
C. Plow back rate +the return on equity (ROE)
D. Plow back rate - the return on equity (ROE)

B&M

24. The growth rate in dividends can be thought of as a sum of two parts. They are:
A. ROE and the Retention Ratio.
B. Dividend yield and growth rate in dividends
C. ROA and ROE
D. Book value per share and EPS

B&M

Dividend Growth Model Formula


12. The required rate of return on the market capitalization rate is estimated as follows:
A. Dividend yield + expected rate of growth in dividends
B. Dividend yield - expected rate of growth in dividends
C. Dividend yield / expected rate of growth in dividends
D. (Dividend yielD. * (expected rate of growth in dividends)

B&M

Capital Asset Pricing Model (CAPM)


49. The acronym CAPM stands for:
A. Capital Asset Pricing Model
B. Certainty Asset Pricing Method

C. Current Arbitrage Pricing Method B & M


D. Cumulative Arbitrage Pricing Model

85

. The capital asset pricing model assumes (E)


a. all investors are price takers.
b. all investors have the same holding period.
c. investors pay taxes on capital gains.
d. both a and b are true.
e. a, b and c are all true.

Bodie

33. If investors do not know their investment horizons for certain (M)
a. the CAPM is no longer valid.
b. the CAPM underlying assumptions are not violated.
Page 15 of 53

MANAGEMENT ADVISORY SERVICES

RISKS

c. the implications of the CAPM are not violated as long as investors liquidity needs re not
priced.
d. the implications of the CAPM are no longer useful.
Bodie
37. The capital asset pricing model (CAPM) states that:
A. The expected risk premium on an investment is proportional to its beta
B. The expected rate of return on an investment is proportional to its beta
C. The expected rate of return on an investment depends on the risk-free rate and the
market rate of return
B&M
D. The expected rate of return on an investment is dependent on the risk-free rate
56. The drawback of the CAPM is that it:
A. Ignores the return on the market portfolio
B. Required a single measure of systematic risk
C. Ignores risk-free return
D. Utilizes too many factors

86

B&M

88

Which statement is not true regarding the Capital Market Line (CML)? (M)
a. The CML is the line from the risk-free rate through the market portfolio.
b. The CML is the best attainable capital allocation line.
c. The CML is also called the security market line.
d. The CML always has a positive slope.

Bodie

. In equilibrium, the marginal price of risk for a risky security must be (M)
a. equal to the marginal price of risk for the market portfolio.
b. greater than the marginal price of risk for the market portfolio.
c. less than the marginal price of risk for the market portfolio.
d. adjusted by its degree of nonsystematic risk.

Bodie

90

. An underpriced security will plot (E)


a. on the Security Market Line.
b. below the Security Market Line.
c. above the Security Market Line.
Bodie
d. either above or below the Security Market Line depending on its covariance with the arket.
e. either above or below the Security Market Line depending on its standard deviation.

B&M

The Security Market Line (SML) is (M)


a. the line that describes the expected return-beta relationship for well-diversified portfolios only.
b. also called the Capital Allocation Line.
c. the line that is tangent to the efficient frontier of all risky assets.
d. the line that represents the expected return-beta relationship.
e. the line that represents the relationship between an individual securitys return and the
markets return.
Bodie

RPCPA, AICPA. CMA & CIA EXAMINATION QUESTIONS

. The security market line (SML) (M)


a. can be portrayed graphically as the expected return-beta relationship.
b. can be portrayed graphically as the expected return-standard deviation of market returns
relationship.
c. provides a benchmark for evaluation of investment performance.
d. a and c.
e. b and c.
Bodie

89

Security Market Line


46. The security market line (SML) shows the relationship between.
A. Expected return and standard deviation
B. Expected return and beta
C. Standard deviation and risk
D. Variance and beta
38. The security market line (SML) is the graph of:
A. Expected rate on investment (Y-axis) vs. variance of return
B. Expected return on investment vs. standard deviation of return
C. Expected rate of return on investment vs. beta
D. A and B

87

47. If a stock is overpriced it would plot:


A. Above the security market line
B. On the security market line

C. Below the security market line


D. On the Y-axis

B&M

Variables
91
. An analysis of a companys planned equity financing using the capital asset pricing model (or
security market line) would incorporate only the
a. Expected market earnings, the current U.S. Treasury bond yield, and the beta coefficient.
b. Expected market earnings and the price-earnings ratio.
c. Current U.S. Treasury bond yield, the price-earnings ratio, and the beta coefficient.
d. Current U.S. Treasury bond yield and the dividend payout ratio.
CMA 1291 1-16
Page 16 of 53

MANAGEMENT ADVISORY SERVICES

RISKS

Formula
92
According to the Capital Asset Pricing Model (CAPM), the expected rate of return on any security
is equal to (M)
a. Rf + [E(RM)].
c. Rf + [E(RM - Rf].
b. Rf + [E(RM) - Rf].
d. E(RM) + Rf.
Bodie
Market Portfolio
93
Which statement is not true regarding the market portfolio? (M)
a. It includes all assets of the universe.
b. It lies on the efficient frontier.
c. All securities in the market portfolio are held in proportion to their market values.
d. It is the tangency point between the capital market line and the indifference curve.
Alpha Coefficient
94
According to the Capital Asset Pricing Model (CAPM), fairly priced securities (M)
a. have positive betas.
c. have negative betas.
b. have zero alphas.
d. have positive alphas.

Bodie

Bodie

D. Variance of the return on the market x Variance of the return on the security Covariance
of the returns on the market and on the security.
Gleim
98

. According to the capital asset pricing model (CAPM), the relevant risk of a security is its
A. Company-specific risk.
C. Systematic risk.
B. Diversifiable risk.
D. Total risk.
CIA 1194 IV-53

39. Beta measures:


A. The ability to diversify risk
B. The change in the rate of return of an investment for a given change in the market rate of
return
C. The actual return on an asset
D. A and C
B&M
40. Beta measure indicates:
A. The ability to diversify risk
B. The change in the rate of return on an investment for a given change in the market return
C. The actual return on an asset
D. A and C
B&M

95

. According to the Capital Asset Pricing Model (CAPM), (M)


a. a security with a positive alpha is considered overpriced.
b. a security with a zero alpha is considered to be a good buy.
c. a security with a negative alpha is considered to be a good buy.
d. a security with a positive alpha is consider to be underpriced.

99

The market risk, beta, of a security is equal to (M)


a. the covariance between the securitys return and the market return divided by the variance of
the market's returns.
b. the covariance between the security and market returns divided by the standard deviation of
the market's returns.
c. the variance of the security's returns divided by the covariance between the security and
market returns.
Bodie
d. the variance of the security's returns divided by the variance of the market's returns.

100

The market portfolio has a beta of (E)


a. 0.
b. 1.

Bodie

Beta Coefficient
96
. A measure that describes the risk of an investment project relative to other investments in
general is the (E)
A. Coefficient of variation.
C. Standard deviation.
B. Beta coefficient.
D. Expected return.
CIA 1187 IV-66
97

. What is the formula for the beta coefficient of a security?


A. Covariance of the returns on the market and on the security Variance of the return on
the market.
B. Covariance of the returns on the market and on the security x Variance of the return on
the market.
C. Variance of the return on the market Variance of the return on the security.

RPCPA, AICPA. CMA & CIA EXAMINATION QUESTIONS

c. -1.
d. 0.5.

Bodie

If the firm is being operated so as to maximize shareholder wealth, and if our basic
assumptions concerning the relationship between risk and return are true, then which of the
following should be true? (M)
a. If the beta of the asset is larger than the firms beta, then the required return on the asset
is less than the required return on the firm.
Page 17 of 53

MANAGEMENT ADVISORY SERVICES


b. If the beta of the asset is smaller than the firms beta, then the required return on the asset
is greater than the required return on the firm.
c. If the beta of the asset is greater than the firms beta prior to the addition of that asset,
then the firms beta after the purchase of the asset will be smaller than the original firms
beta.
d. If the beta of an asset is larger than the firms beta prior to the addition of that asset, then
the required return on the firm will be greater after the purchase of that asset than prior to
its purchase.
Brigham
.

Which of the following statements is most correct? (M)


a. Beta measures market risk, but if a firms stockholders are not well diversified, beta may
not accurately measure stand-alone risk.
b. If the calculated beta underestimates the firms true investment risk, then the CAPM
method will overestimate ks.
c. The discounted cash flow method of estimating the cost of equity cant be used unless the
growth component, g, is constant during the analysis period.
Brigham
d. An advantage shared by both the DCF and CAPM methods of estimating the cost of
equity capital, is that they yield precise estimates and require little or no judgement.

Which of the following statements is correct? (M)


a. The cost of capital used to evaluate a project should be the cost of the specific type of
financing used to fund that project.
b. The cost of debt used to calculate the weighted average cost of capital is based on an
average of the cost of debt already issued by the firm and the cost of new debt.
c. One problem with the CAPM approach to estimating the cost of equity capital is that if a
firms stockholders are, in fact, not well diversified, beta may be a poor measure of the
firms true investment risk.
d. The bond-yield-plus-risk-premium approach is the most sophisticated and objective
method of estimating a firms cost of equity capital.
e. The cost of equity capital is generally easier to measure than the cost of debt, which
varies daily with interest rates, or the cost of preferred stock since preferred stock is
issued infrequently.
Brigham

101

. In the context of the Capital Asset Pricing Model (CAPM) the relevant measure of risk is (E)
a. unique risk.
c. standard deviation of returns.
b. beta.
d. variance of returns.
Bodie

RISKS
A.
B.
C.
D.

Increase 20% faster than the market in up markets


Increase 20% faster than the market in down markets
Increase 120% faster than the market in up markets
Increase 120% faster than the market in down markets

Risk-free Rate
102
. What is the expected return of a zero-beta security? (M)
a. The market rate of return.
c. A negative rate of return.
b. Zero rate of return.
d. The risk-free rate.

B&M

Bodie

Risk Premium
103
. According to the CAPM, the risk premium an investor expects to receive on any stock or portfolio
increases: (E)
a. directly with alpha.
d. inversely with beta.
b. inversely with alpha.
e. in proportion to its standard deviation.
c. directly with beta.
Bodie
104

. The risk premium on the market portfolio will be proportional to (M)


a. the average degree of risk aversion of the investor population.
b. the risk of the market portfolio as measured by its variance.
c. the risk of the market portfolio as measured by its beta.
d. both a and b are true.
e. both a and c are true.

Bodie

36. The market risk premium is:


A. The difference between the rate of return on an asset and the risk-free rate
B. The difference between the rate of return on the market portfolio and the risk-free rate
C. The risk-free rate
D. The market rate of return
B&M
Pure Play Method
. Which of the following methods involves calculating an average beta for firms in a similar
business and then applying that beta to determine a projects beta? (M)
a. Risk premium method.
c. Accounting beta method.
b. Pure play method.
d. CAPM method.
Brigham
105

45. A stock with a beta of 1.2 would be expected to:


RPCPA, AICPA. CMA & CIA EXAMINATION QUESTIONS

. Interstate Transport has a target capital structure of 50 percent debt and 50 percent common
equity. The firm is considering a new independent project that has an expected return of 13
Page 18 of 53

MANAGEMENT ADVISORY SERVICES


percent and is not related to transportation. However, a pure play proxy firm has been
identified that is exclusively engaged in the new line of business. The proxy firm has a beta of
1.38. Both firms have a marginal tax rate of 40 percent, and Interstates before-tax cost of debt
is 12 percent. The risk-free rate is 10 percent and the market risk premium is 5 percent. The
firm should(M)
a. Reject the project; its expected return is less than the firms required rate of return on the
project of 16.9 percent.
b. Accept the project; its expected return is greater than the firms required rate of return on
the project of 12.05 percent.
c. Reject the project; its expected return is only 13 percent.
d. Accept the project; its expected return exceeds the risk-free rate and the before-tax cost
of debt.
e. Be indifferent between accepting or rejecting; the firms required rate of return on the
project equals its expected return.
Brigham
Required Rate of Return
106
. If the return on the market portfolio is 10% and the risk-free rate is 5%, what is the effect on a
company's required rate of return on its stock of an increase in the beta coefficient from 1.2 to
1.5?
A. 3% increase
C. No change
B. 1.5% increase
D. 1.5% decrease
Gleim
Expected Rate of Return vs. Required Rate of Return
107
. Security X has an expected rate of return of 0.11 and a beta of 1.5. The risk-free rate is 0.05 and
the market expected rate of return is 0.09. According to the Capital Asset Pricing Model, this
security is (M)
a. underpriced.
b. overpriced.
c. fairly priced.
d. cannot be determined from data provided.
Bodie
108

. The risk-free rate is 7 percent. The expected market rate of return is 15 percent. If you expect
stock X with a beta of 1.3 to offer a rate of return of 12 percent, you should (M)
a. buy stock X because it is overpriced.
b. sell short stock X because it is overpriced.
c. sell stock short X because it is underpriced.
d. buy stock X because it is underpriced.
Bodie

RPCPA, AICPA. CMA & CIA EXAMINATION QUESTIONS

RISKS
109

. Given the following two stocks A and B


Expected rate of return
Beta
A
0.12
1.2
B
0.14
1.8
If the expected market rate of return is 0.09 and the risk-free rate is 0.05, which security would
be considered the better buy and why? (M)
a. A because it offers an expected excess return of 1.2%.
b. B because it offers an expected excess return of 1.8%.
c. A because it offers an expected excess return of 2.2%.
d. B because it offers an expected return of 14%.
e. B because it has a higher beta.
Bodie

Weighted-Average Cost of Capital (WACC)


1. The cost of capital is defined as (E)
a. the simple average of the interest rates of all debt outstanding.
b. the simple average of the cost of debt and equity.
c. the weighted average of the interest rates of all debt outstanding.
d. the weighted average of the cost of debt and equity.
.

S, S & S

Which of the following is not considered a capital component for the purpose of calculating the
weighted average cost of capital as it applies to capital budgeting? (E)
a. Long-term debt.
c. Short-term debt.
b. Common stock.
d. Preferred stock.
Brigham

28. The combined weighted average interest rate that a firm incurs on its long-term debt, preferred
stock, and common stock is the
a. cost of capital.
c. cutoff rate.
b. discount rate.
d. internal rate of return.
Barfield
108.The weighted average cost of capital represents the
a. cost of bonds, preferred stock, and common stock divided by the three sources.
b. equivalent units of capital used by the organization.
c. overall cost of capital from all organization financing sources.
d. overall cost of dividends plus interest paid by the organization.

Barfield

29. The weighted average cost of capital that is used to evaluate a specific project should be
based on the
a. mix of capital components that was used to finance a project from last year.
Page 19 of 53

MANAGEMENT ADVISORY SERVICES


b. overall capital structure of the corporation.
c. cost of capital for other corporations with similar investments.
d. mix of capital components for all capital acquired in the most recent fiscal year.

RISKS

Barfield

110

. Which of the following statements is correct? (M)


a. Because we often need to make comparisons among firms that are in different income tax
brackets, it is best to calculate the WACC on a before-tax basis.
b. If a firm has been suffering accounting losses and is expected to continue suffering such
losses, and therefore its tax rate is zero, it is possible that its after-tax component cost of
preferred stock as used to calculate the WACC will be less than its after-tax component
cost of debt.
c. Normally, the cost of external equity raised by issuing new common stock is above the
cost of retained earnings. Moreover, the higher the growth rate is relative to the dividend
yield, the more the cost of external equity will exceed the cost of retained earnings.
d. The lower a companys tax rate, the greater the advantage of using debt in terms of
lowering its WACC.
Brigham

111

. When calculating a firm's cost of capital, all of the following are true except that (E)
A. The cost of capital of a firm is the weighted average cost of its various financing
components.
B. The calculation of the cost of capital should focus on the historical costs of alternative
forms of financing rather than market or current costs.
C. All costs should be expressed as after-tax costs.
D. The time value of money should be incorporated into the calculations.
CMA 1288 1-2

112

. A company has made the decision to finance next year's capital projects through debt rather
than additional equity. The benchmark cost of capital for these projects should be (M)
A. The before-tax cost of new-debt financing.
B. The after-tax cost of new-debt financing.
C. The cost of equity financing.
D. The weighted-average cost of capital.
CIA 0597 IV-42

113

. Which of the following is true regarding the calculation of a firm's cost of capital? (E)
A. The cost of capital of a firm is the weighted-average cost of its various financing
components.
B. All costs should be expressed as pre-tax costs.
C. The time value of money should be excluded from the calculations.
D. The cost of capital is the cost of equity.
CMA 1288 1-2

RPCPA, AICPA. CMA & CIA EXAMINATION QUESTIONS

18. The weighted-average cost of capital approach to decision making is not directly affected by
the: (E)
A. proposed mix of debt, equity, and existing funds used to implement the project
B. value of the common stock
C. cost of debt outstanding
D. current budget for expansion
AICPA adapted
30. Debt in the capital structure could be treated as if it were common equity in computing the
weighted average cost of capital if the debt were
a. callable.
c. cumulative.
b. participating.
d. convertible.
Barfield
.

For a typical firm with a given capital structure, which of the following is correct? (Note: All
rates are after taxes.) (E)
a. kd > ke > ks > WACC.
c. WACC > ke > ks > kd.
b. ks > ke > kd > WACC.
d. ke > ks > WACC > kd.
Brigham

33. Which of the following is true?


A. rD < rA < rE
B. rE < rD < rA

C. rE < rA < rD
D. None of the above are true

B&M

31. Generally, which of the following is true?


A. rD > rA > rE
B. rE > rD > rA

C. rE > rA > rD
D. None of the above are true

B&M

114

. Which of the following statements is correct? (M)


a. The WACC should include only after-tax component costs. Therefore, the required rates
of return (or market rates) on debt, preferred, and common equity (kd, kp, and ks) must be
adjusted to an after-tax basis before they are used in the WACC equation.
Brigham
b. The cost of retained earnings is generally higher than the cost of new common stock.
c. Preferred stock is riskier to investors than is debt. Therefore, if someone told you that the
market rates showed kd > kp for a given company, that person must have made a mistake.
d. If a company with a debt ratio of 50 percent were suddenly exempted from all future
income taxes, then, all other things held constant, this would cause its WACC to increase.

115

. Which of the following statements is most correct? (M)


Page 20 of 53

MANAGEMENT ADVISORY SERVICES


a. The weighted average cost of capital for a given capital budget level is a weighted
average of the marginal cost of each relevant capital component that makes up the firms
target capital structure.
b. The weighted average cost of capital is calculated on a before-tax basis.
c. An increase in the risk-free rate is likely to increase the marginal costs of both debt and
equity financing.
d. Statements a and c are correct.
Brigham

RISKS
c. The cost of retained earnings is generally more expensive than the cost of issuing new
common stock, because it includes an opportunity cost.
d. Statements a and b are correct.
e. All of the statements above are correct.
Brigham
120

. Which of the following statements about the cost of capital is incorrect? (E)
a. A companys target capital structure affects its weighted average cost of capital.
b. Weighted average cost of capital calculations should be based on the after-tax costs of all
the individual capital components.
c. If a companys tax rate increases, then, all else equal, its weighted average cost of capital
will increase.
d. Flotation costs can increase the weighted average cost of capital.
e. An increase in the risk-free rate is likely to increase the marginal costs of both debt and
equity financing.
Brigham

116

. A company has a capital structure that consists of 50 percent debt and 50 percent equity.
Which of the following statements is most correct? (E)
a. The cost of equity financing is greater than or equal to the cost of debt financing.
b. The WACC exceeds the cost of equity financing.
c. The WACC is calculated on a before-tax basis.
d. The WACC represents the cost of capital based on historical averages. In that sense, it
does not represent the marginal cost of capital.
e. The cost of retained earnings exceeds the cost of issuing new common stock.
Brigham

117

. Which of the following statements is most correct? (E)


a. The WACC is a measure of the before-tax cost of capital.
b. Typically the after-tax cost of debt financing exceeds the after-tax cost of equity financing.
c. The WACC measures the marginal after-tax cost of capital.
d. Statements a and b are correct.
e. Statements b and c are correct.
Brigham

A company estimates that an average-risk project has a WACC of 10 percent, a belowaverage risk project has a WACC of 8 percent, and an above-average risk project has a
WACC of 12 percent. Which of the following independent projects should the company
accept? (E)
a. Project A has average risk and a return of 9 percent.
b. Project B has below-average risk and a return of 8.5 percent.
c. Project C has above-average risk and a return of 11 percent.
d. All of the projects above should be accepted.
e. None of the projects above should be accepted.
Brigham

118

. Campbell Co. is trying to estimate its weighted average cost of capital (WACC). Which of the
following statements is most correct? (E)
a. The after-tax cost of debt is generally cheaper than the after-tax cost of preferred stock.
b. Since retained earnings are readily available, the cost of retained earnings is generally
lower than the cost of debt.
c. If the companys beta increases, this will increase the cost of equity financing, even if the
company is able to rely on only retained earnings for its equity financing.
d. Statements a and b are correct.
e. Statements a and c are correct.
Brigham

119

. Which of the following statements is most correct? (E)


a. The WACC represents the after-tax cost of capital.
b. The WACC represents the marginal cost of capital.

RPCPA, AICPA. CMA & CIA EXAMINATION QUESTIONS

22. Which of the following regarding the weighted-average cost of capital is true?
a. The tax effect of preferred stock dividends should be included in the calculation of
weighted-average cost of capital.
b. The tax effect of common stock dividends should be included in the calculation of
weighted-average cost of capital.
c. The tax effect of debt should be included in the calculation of the weighted-average cost
of capital.
d. Taxes do not affect the weighted-average cost of capital.
S, S & S
23. Which of the following is true regarding the weighted-average cost of capital?
a. A company may have two weighted-average costs of capital if the firm's capital structure is
so large that new common stock must be sold.
Page 21 of 53

MANAGEMENT ADVISORY SERVICES

RISKS

b. The book value of the components of capital should always be used to calculate the
weighted-average cost of capital.
c. The cost of common equity is lower than the cost of retained earnings.
d. The cost of preferred stock is adjusted for the tax deduction associated with preferred
dividends.
S, S & S
WACC in a Tax-Free Environment
21. The cost of capital for a firm, rWACC, in a tax free environment is:
A. Equal to the expected EBIT divided by market value of the unlevered firm
B. Equal to rA, the rate of return for that business risk class
C. Equal to the overall rate of return required on the levered firm
D. All of the above

B&M

Capital Asset Pricing Model


121
. Wyden Brothers has no retained earnings. The company uses the CAPM to calculate the cost
of equity capital. The companys capital structure consists of common stock, preferred stock,
and debt. Which of the following events will reduce the companys WACC? (M)
a. A reduction in the market risk premium.
b. An increase in the flotation costs associated with issuing new common stock.
c. An increase in the companys beta.
d. An increase in expected inflation.
e. An increase in the flotation costs associated with issuing preferred stock.
Brigham
Required Rate of Return
122
. Louisiana Enterprises, an all-equity firm, is considering a new capital investment. Analysis has
indicated that the proposed investment has a beta of 0.5 and will generate an expected return
of 7 percent. The firm currently has a required return of 10.75 percent and a beta of 1.25. The
investment, if undertaken, will double the firms total assets. If k RF is 7 percent and the market
return is 10 percent, should the firm undertake the investment? (Choose the best answer.) (D)
a. Yes; the expected return of the asset (7%) exceeds the required return (6.5%).
b. Yes; the beta of the asset will reduce the risk of the firm.
c. No; the expected return of the asset (7%) is less than the required return (8.5%).
d. No; the risk of the asset (beta) will increase the firms beta.
e. No; the expected return of the asset is less than the firms required return, which is
10.75%.
Brigham

RPCPA, AICPA. CMA & CIA EXAMINATION QUESTIONS

123

. Interstate Transport has a target capital structure of 50 percent debt and 50 percent common
equity. The firm is considering a new independent project that has a return of 13 percent and
is not related to transportation. However, a pure play proxy firm has been identified that is
exclusively engaged in the new line of business. The proxy firm has a beta of 1.38. Both firms
have a marginal tax rate of 40 percent, and Interstates before-tax cost of debt is 12 percent.
The risk-free rate is 10 percent and the market risk premium is 5 percent. The firm should(M)
a. Reject the project; its return is less than the firms required rate of return on the project of
16.9 percent.
b. Accept the project; its return is greater than the firms required rate of return on the project
of 12.05 percent.
c. Reject the project; its return is only 13 percent.
d. Accept the project; its return exceeds the risk-free rate and the before-tax cost of debt.
e. Be indifferent between accepting or rejecting; the firms required rate of return on the
project equals its expected return.
Brigham

Optimal Project Selection


124
. Jackson Corporation is evaluating the following four independent, investment opportunities:
Project
Cost
Expected Rate of Return
A
$300,000
14%
B
150,000
10
C
200,000
13
D
400,000
11
Jacksons target capital structure is 60 percent debt and 40 percent equity. The yield to
maturity on the companys debt is 10 percent. Jackson will incur flotation costs for a new
equity issuance of 12 percent. The growth rate is a constant 6 percent. The stock price is
currently $35 per share for each of the 10,000 shares outstanding. Assume that the firm has
no retained earnings. If the companys tax rate is 30 percent, then which of the projects will be
accepted? (M)
a. Project A.
c. Projects A, C, and D.
Brigham
b. Projects A and C.
d. All of the investment projects will be taken.
Sensitivity Analysis
Risk, Required Return and project betas
. If the firm is being operated so as to maximize shareholder wealth, and if our basic
assumptions concerning the relationship between risk and return are true, then which of the
following should be true? (M)
a. If the beta of the asset is larger than the firms beta, then the required return on the asset
is less than the required return on the firm.
Page 22 of 53

MANAGEMENT ADVISORY SERVICES

RISKS

b. If the beta of the asset is smaller than the firms beta, then the required return on the asset
is greater than the required return on the firm.
c. If the beta of the asset is greater than the corporate beta prior to the addition of that asset,
then the corporate beta after the purchase of the asset will be smaller than the original
corporate beta.
d. If the beta of an asset is larger than the corporate beta prior to the addition of that asset,
then the required return on the firm will be greater after the purchase of that asset than
prior to its purchase.
Brigham

b. Managers attitudes toward risk differ and some managers may set a target capital
structure other than the one that would maximize stock price.
c. Managers often have a responsibility to provide continuous service; they must preserve
the long-run viability of the enterprise. Thus, the goal of employing leverage to maximize
short-run stock price and minimize capital cost may conflict with long-run viability.
d. All of the statements above are correct.
e. None of the statements above represents a serious impediment to the practical
application of leverage analysis in capital structure determination.
Brigham

Beta and project risk


. Which of the following statements is correct? (D)
a. A relatively risky future cash outflow should be evaluated using a relatively low discount
rate.
b. If a firms managers want to maximize the value of the stock, they should concentrate
exclusively on projects market, or beta, risk.
c. If a firm evaluates all projects using the same cost of capital, then the riskiness of the firm
as measured by its beta will probably decline over time.
d. If a firm has a beta that is less than 1.0, say 0.9, this would suggest that its assets returns
are negatively correlated with the returns of most other firms assets.
Brigham

Operating Leverage
125
. Which class of leverage causes earnings before interest and taxes to be more sensitive to
changes in sales? (M)
A. Credit.
C. Operating.
B. Financial.
D. Intrinsic.
CIA 0593 IV-57

Comprehensive
. Which of the following statements is correct? (M)
a. Although some methods of estimating the cost of equity capital encounter severe
difficulties, the CAPM is a simple and reliable model that provides great accuracy and
consistency in estimating the cost of equity capital.
b. The DCF model is preferred over other models to estimate the cost of equity because of
the ease with which a firms growth rate is obtained.
c. The bond-yield-plus-risk-premium approach to estimating the cost of equity is not always
accurate but its advantages are that it is a standardized and objective model.
d. Depreciation-generated funds are an additional source of capital and, in fact, represent
the largest single source of funds for some firms.
Brigham
LEVERAGE
Limits of leverage
21. Which of the following are practical difficulties associated with capital structure and degree of
leverage analyses? (M)
a. It is nearly impossible to determine exactly how P/E ratios or equity capitalization rates (k s
values) are affected by different degrees of financial leverage.
RPCPA, AICPA. CMA & CIA EXAMINATION QUESTIONS

Page 23 of 53

MANAGEMENT ADVISORY SERVICES

RISKS

126

. The percentage change in earnings before interest and taxes associated with the percentage
change in sales volume represents the degree of (E)
A. Operating leverage.
C. Breakeven leverage.
B. Financial leverage.
D. Combined leverage.
CIA 1189 IV-54

a. Trading in the stock of other companies


b. Retiring bonds before maturity

c. Using funds provided by creditors


d. Purchasing and selling treasury stocks

127

. The degree of operating leverage (DOL) is


A. Constant at all levels of sales.
B. A measure of the change in earnings available to common stockholders associated with a
given change in operating earnings.
C. A measure of the change in operating income resulting from a given change in sales.
D. Lower if the degree of total leverage is higher, other things held constant. CIA 0594 IV-52

128

. Companies experience changes in interest expenses, variable cost per unit, quantity of units
sold, and fixed costs. Their degree of operating leverage is not affected by the change in
A. Interest expenses.
C. Quantity of units sold.
B. Variable cost per unit.
D. Fixed costs.
CIA 1193 IV-53

129

. A firm with a higher degree of operating leverage when compared to the industry average
implies that the
A. Firm has higher variable costs.
B. Firm's profits are more sensitive to changes in sales volume.
C. Firm is more profitable.
D. Firm is less risky.
CMA 0695 1-1

Financial Leverage
*. Securing the funds for investment at a fixed rate of return to fund suppliers, to enhance the
well-being of the common stockholders is known as
a. Financial leverage.
c. Prudent borrowing.
b. Fund management.
d. Financial arbitrage.
RPCPA 1097
*.

It refers to the practice of financing assets with borrowed capital. Its extensive use may impact
on the return on common stockholders equity to be above or below the rate of return on total
assets.
a. Discounting.
c. Leverage.
b. Mortgage.
d. Arbitrage.
RPCPA 0597

*.

Trading on equity pertains to the practice of (E)

RPCPA, AICPA. CMA & CIA EXAMINATION QUESTIONS

RPCPA 1077, 0588


Page 24 of 53

MANAGEMENT ADVISORY SERVICES


*.

This accounting terminology has reference to long-term debt and means that you borrow
somebodys money at an interest rate which is lower than the rate which you can earn on that
money.
a. Pooling of interest.
c. Kiting.
b. Leverage or trading on equity.
d. None of the above.
RPCPA 1084

RISKS
*.

The use of borrowed capital by business firms is referred to as leverage or trading on equity.
This leverage is likely to be a sound financial strategy for stockholders of companies having
a. Cyclical high and low amounts of reported earnings.
b. Steady amounts of reported earnings.
c. Volatile fluctuation in reported earnings over short periods of time.
d. Steadily declining amounts of reported earnings.
RPCPA 1079

130

. The purchase of treasury stock with a firm's surplus cash


CMA 1291 1-5
A. Increases a firm's assets.
C. Increases a firm's interest coverage ratio.
B. Increases a firm's financial leverage.
D. Dilutes a firm's earnings per share.

131

. When a company increases its degree of financial leverage (DFL)


a. The equity beta of the company falls.
b. The systematic risk of the company falls.
c. The systematic risk of the company rises.
d. The standard deviation of returns on the equity of the company rises.

CIA 0597 IV-50

132

. Sylvan Corporation has the following capital structure.


Debenture bonds
$10,000,000
Preferred equity
1,000,000
Common equity
39,000,000
The financial leverage of Sylvan Corporation would increase as a result of (M)
A. Issuing common stock and using the proceeds to retire preferred stock.
B. Maintaining the same dollar level of cash dividends as the prior year, even though
earnings have increased by 7%.
C. Financing its future investments with a higher percentage of bonds.
D. Financing its future investments with a higher percentage of equity funds. CMA 0690 1-9

133

. Everything else being equal, a <List A> highly leveraged firm will have <List B> earnings per
share. (D)
CIA 0595 IV-51
A.
B.
C.
D.
List A
More
More
Less
Less
List B
Lower
Less volatile
Less volatile
Higher

134

. A company is considering the early retirement of its 10%, 10-year bonds payable. Before
retiring the bonds, the company's capital structure was
Current liabilities
$125,000

RPCPA, AICPA. CMA & CIA EXAMINATION QUESTIONS

Page 25 of 53

MANAGEMENT ADVISORY SERVICES


Long-term liabilities:
Notes payable (due in 5 years)
Bonds payable
Premium on bonds payable
Owner's equity:
Common shares ($5 par value)
Share premium in excess of par
Retained earnings
If the bonds can be retired at 103.5%, the
A. Debt-equity ratio will increase.
B. Financial leverage will decrease.

RISKS
137

200,000
300,000
25,000
150,000
50,000
450,000
CIA 1193 IV-48
C. Asset turnover ratio will decrease.
D. Return on owner's equity will decrease.

Financial leverage and EPS


135
. Volga Publishing is considering a proposed increase in its debt ratio, which will also increase
the companys interest expense. The plan would involve the company issuing new bonds and
using the proceeds to buy back shares of its common stock. The companys CFO expects that
the plan will not change the companys total assets or operating income. However, the
companys CFO does estimate that it will increase the companys earnings per share (EPS).
Assuming the CFOs estimates are correct, which of the following statements is most correct?
(E)
a. Since the proposed plan increases Volgas financial risk, the companys stock price still
might fall even though its EPS is expected to increase.
b. If the plan reduces the companys WACC, the companys stock price is also likely to
decline.
c. Since the plan is expected to increase EPS, this implies that net income is also expected
to increase.
d. Statements a and b are correct.
e. Statements a and c are correct.
Brigham

. Company A and Company B have the same tax rate, the same total assets, and the same
basic earning power. Both companies have a basic earning power that exceeds their beforetax costs of debt, kd. However, Company A has a higher debt ratio and higher interest expense
than Company B. Which of the following statements is most correct? (E)
a. Company A has a lower net income than B.
b. Company A has a lower ROA than B.
c. Company A has a lower ROE than B.
d. Statements a and b are correct.
Brigham

138

. Firm U and Firm L each have the same total assets. Both firms also have a basic earning
power of 20 percent. Firm U is 100 percent equity financed, while Firm L is financed with 50
percent debt and 50 percent equity. Firm Ls debt has a before-tax cost of 8 percent. Both
firms have positive net income. Which of the following statements is most correct? (E)
a. The two companies have the same times interest earned (TIE) ratio.
b. Firm L has a lower ROA.
c. Firm L has a lower ROE.
d. Statements a and b are correct.
e. Statements b and c are correct.
Brigham

Financial leverage and ratios


139
. Company A and Company B have the same total assets, operating income (EBIT), tax rate,
and business risk. Company A, however, has a much higher debt ratio than Company B.
Company As basic earning power (BEP) exceeds its cost of debt financing (kd). Which of the
following statements is most correct? (M)
a. Company A has a higher return on assets (ROA) than Company B.
b. Company A has a higher times interest earned (TIE) ratio than Company B.
c. Company A has a higher return on equity (ROE) than Company B, and its risk, as
measured by the standard deviation of ROE, is also higher than Company Bs.
d. Statements b and c are correct.
Brigham

136

. Which of the following statements is most correct? (E)


a. Increasing financial leverage is one way to increase a firms basic earning power (BEP).
b. Firms with lower fixed costs tend to have greater operating leverage.
c. The debt ratio that maximizes EPS generally exceeds the debt ratio which maximizes
share price.
d. Statements a and b are correct.
e. Statements a and c are correct.
Brigham

Financial leverage and ratios


RPCPA, AICPA. CMA & CIA EXAMINATION QUESTIONS

Operating and Financial Leverage


20. Which of the following statements is most correct? (M)
a. Firms whose sales are very sensitive to changes in the business cycle are more likely to
rely on debt financing.
b. Firms with large tax loss carry forwards are more likely to rely on debt financing.
c. Firms with a high operating leverage are more likely to rely on debt financing.
d. Statements a and c are correct.
e. None of the statements above is correct.
Brigham
Page 26 of 53

MANAGEMENT ADVISORY SERVICES

RISKS
B. Endowment.

Comprehensive
140
. Which of the changes in leverage would apply to a company that substantially increases its
investment in fixed assets as a proportion of total assets and replaces some of its long-term
debt with equity? (M)
CIA 0590 IV-56
A.
B.
C.
D.
Financial Leverage
Increase
Decrease
Increase
Decrease
Operating Leverage
Decrease
Increase
Increase
Decrease
INSURANCE
141
. Insurance may best be defined as
A. A system for transferring risk through risk avoidance or loss control.
B. Any contract that conveys an insurable interest.
C. A form of pure risk called gambling.
Gleim
D. A means of combining many loss exposures so that losses are shared by all participants.

D. Term.

Gleim

146

. Jon Berstock is an employee of PR, Inc. During his employment, the corporation's earnings
have doubled, largely because of Jon's ability to attract new accounts. PR therefore insured
his life for a substantial sum. If Jon dies, will PR be able to collect the insurance proceeds?
A. Yes, because a corporation has an insurable interest in all its employees since it can act
only through agents.
B. Yes, because PR has a pecuniary interest in Jon's continued life.
C. No, because PR will continue to exist after Jon's death.
D. No, because Jon was not a stockholder or officer of PR, Inc.
Gleim

147

. Which of the following is a characteristic of fire insurance?


A. It is more standardized than life insurance.
B. It is written for a relatively short period but usually includes an incontestability clause.
C. A policy must be valued and contain a pro rata clause.
D. The insurable interest must be an ownership interest in the property itself.
Gleim

142

. Which of the following is the best functional definition of insurance?


A. A legal contract by which the insurer, in return for consideration, agrees to pay another
person if a stated loss or injury occurs.
B. A legal contract by which an insurance company, in return for premiums, agrees to pay the
policyholder if a certain event occurs.
C. A written promise by the insurer to pay the beneficiary if loss occurs from the occurrence
of a contingent event.
D. A writing issued by an insurance company, for a consideration, that promises to indemnify
a beneficiary for a loss from an existing risk or one which arises later.
Gleim

143

. In which way does the formation of an insurance contract differ from any other contract?
A. The requirement that the insured must have an insurable interest.
B. The insurance contract is not valid unless written.
C. Consideration is not needed for the formation of an insurance contract.
D. In insurance, only the insured can commit a breach.
Gleim

144

. Life insurance
A. Is a contract of indemnity.
B. Usually has short-term policies.

C. Covers only the mortality risk.


D. Generally has no cash value.

Gleim

145

. Life insurance is offered in several forms. The kind that offers no investment feature is
A. Whole life.
C. Straight life.
RPCPA, AICPA. CMA & CIA EXAMINATION QUESTIONS

Page 27 of 53

MANAGEMENT ADVISORY SERVICES

RISKS

148

. The typical life insurance policy contains


A. No exclusion for death during military service.
B. A clause allowing coverage for death during noncommercial flight.
C. A prohibition on reinstatement.
D. A provision for a grace period for premium payment.

Gleim

B. Jewelry, Inc. will recover $2,000, the loss less the amount in excess of the $10,000
display limitation.
C. Jewelry, Inc. will recover the full $4,000 since the warranty will be construed as a mere
representation.
D. Jewelry, Inc. will recover the full $4,000 since attaching the application to the policy is
insufficient to make it a part thereof.
AICPA 1181 L-55

149

. A fire insurance policy ordinarily indemnifies for losses arising from


A. Friendly, but not hostile, fires.
B. Hostile, but not friendly, fires.
C. Both hostile and friendly fires.
D. Smoke produced by friendly or hostile fires.

Gleim

150

. Which of the following wrongful acts prevents recovery under a policy of fire insurance?
A. Arson by the insured's employees or agents.
B. Arson by third persons unrelated to the insured.
C. An act by the insured intended to cause the damage.
D. Gross negligence but not amounting to recklessness and willful misconduct.
Gleim

151

. Which of the following is not a type of insurance policy that provides liability coverage?
A. Malpractice insurance.
C. Automobile insurance.
B. Homeowners insurance.
D. Fire insurance.
Gleim

152

. The purpose of a co-insurance clause is to


A. Encourage policyholders to bear a proportionate part of any loss.
B. Encourage insurers to pay the face amount of the policy in the event of a partial loss on
the part of the insured.
C. Encourage policyholders to insure commercial property for an amount that is near to its
full replacement cost.
D. Encourage policyholders to insure commercial property for an amount that is significantly
less than its full replacement cost.
Gleim

153

. Jewelry, Inc. took out an insurance policy with Insurance Company which covered the stock of
jewelry. Insurance agreed to indemnify for losses due to theft of the jewels displayed. The
application contained the following provision: "It is hereby warranted that the maximum value
of the jewelry displayed shall not exceed $10,000." The insurance policy's coverage was for
$8,000. Subsequently, thieves smashed the store window and stole $4,000 worth of jewels
when the total value of the display was $12,000. Which of the following is correct?
A. Jewelry, Inc. will recover nothing.

RPCPA, AICPA. CMA & CIA EXAMINATION QUESTIONS

Page 28 of 53

MANAGEMENT ADVISORY SERVICES

RPCPA, AICPA. CMA & CIA EXAMINATION QUESTIONS

RISKS

Page 29 of 53

MANAGEMENT ADVISORY SERVICES

RISKS
ANSWER EXPLANATIONS

Investment Risk
Financial Risk
Optimal Capital Structure
Cost of Capital
Debt Capital vs. Equity Capital
Imputed Costs vs. Explicit Costs
Cost of Debt Capital
Marginal Cost of Debt
Cost of Equity Capital
Marginal Cost of Capital
Optimal Cost of Capital
Dividend Growth Model
Capital Asset Pricing Model (CAPM)
Leverage
Probability & Statistics

RPCPA, AICPA. CMA & CIA EXAMINATION QUESTIONS

Page 30 of 53

. Answer (A) is correct. Investment risk is analyzed in terms of the probability that the actual return on an investment
will be lower than the expected return. Comparing a project's expected return to the return on an asset of similar risk,
helps determine whether the project is worth investing in. If the expected return on a project exceeds the return on an
asset of comparable risk, then the project should be pursued.
Answer (B) is incorrect because a project should be pursued only if its expected return exceeds the return on
investments of similar risk. Answer (C) is incorrect because a project should be pursued only if its expected return
exceeds the return on investments of similar risk. Answer (D) is incorrect because a project should be pursued only if its
expected return exceeds the return on investments of similar risk.

. Answer (D) is correct. Interest-rate risk is the risk of fluctuations in the value of an asset due to changes in interest
rates. One component of interest-rate risk is price risk, which is portrayed as a decline in the value of bonds as interest
rates increase. Reinvestment-rate risk is another component of interest-rate risk. If interest rates decline, lower returns
will be available for reinvestment of interest and principal payments received.
Answer (A) is incorrect because purchasing-power risk concerns inflation, and default risk concerns nonpayment by the
debtor. Answer (B) is incorrect because market risk concerns price changes in the overall securities markets. Answer (C)
is incorrect because portfolio risk is the risk remaining in a portfolio after diversifying investments.

. REQUIRED: The marketable securities with the least default risk.


DISCUSSION: (B) The marketable securities with the lowest default risk are those issued by the federal government
because they are backed by the full faith and credit of the U.S. government an are therefore the least risky form of
investment.
Answer (A) is incorrect because securities issued by a federal agency are first backed by that agency and secondarily
by the U.S. government. Agency securities are issued by agencies and corporations created by the federal government,
such as the Federal Housing Administration. Answer (C) is incorrect because repurchase agreements could become
worthless if the organization agreeing to make the repurchase goes bankrupt. Answer (D) is incorrect because
commercial paper is unsecured.

. Answer (B) is correct. Prices of all stocks, even the value of portfolios, are correlated to some degree with broad
swings in the stock market. Market risk is the risk that changes in a stock's price will result from changes in the stock
market as a whole. Market risk is commonly referred to as nondiversifiable risk.
Answer (A) is incorrect because purchasing-power risk is the risk that a general rise in the price level will reduce the
quantity of goods that can be purchased with a fixed sum of money. Answer (C) is incorrect because nonmarket risk is
the risk that is influenced by an individual firm's policies and decisions. Nonmarket risk is diversifiable because it is
specific to each firm. Answer (D) is incorrect because interest-rate risk is the risk that the value of an asset will fluctuate
due to changes in the interest rate.

. Answer (D) is correct. Nonsystematic risk is considered to be unique to a firm and therefore random. In capital
market theory, nonsystematic risk can largely be avoided through diversified investments.
Answers (A) and (B) are incorrect because nonsystematic risk cannot be correlated with any variable, whether
qualitative or quantitative. Answer (C) is incorrect because may be reduced through diversification.

6
7

.
.

Corporate risk
Corporate risk

Answer: b Diff: E
Answer: e Diff: M

. Answer (B) is correct. Total risk is the risk of a single asset, whereas market risk is its risk if it is held in a large
portfolio of diversified securities. Total risk therefore includes diversifiable and undiversifiable risk.
Answer (A) is incorrect because nonmarket risk is diversifiable and undiversifiable risk. Answer (C) is incorrect because
portfolio risk is diversifiable and undiversifiable risk. Answer (D) is incorrect because market risk is diversifiable and
undiversifiable risk.

. Answer (D) is correct. The greater the standard deviation of returns, the greater the risk is for an asset. The
expected return can vary anywhere between the large standard deviation of returns, creating the risk that the actual
return is significantly low in the range of the standard deviation of returns.

Answer (A) is incorrect because an asset with high risk will have a high expected return to compensate for the additional
risk. Answer (B) is incorrect because an asset with high risk will have a higher price than an asset with low risk due to
the high expected return. Answer (C) is incorrect because an asset with high risk will have a constant expected return,
not an increasing expected return.
10

. REQUIRED: The circumstance in which overall company risk will be lowest.


DISCUSSION: (A) A common general definition is that risk is an investment with an unknown outcome but a known
probability distribution of returns (a known mean and standard deviation). An increase in the standard deviation
(variability) of returns is synonymous with an increase in the riskiness of a project. Risk is also increased when the
projects returns are positively (directly) correlated with other investments in the firms portfolio, i.e, risk increases when
returns on all projects rise or fall together. Consequently, the overall risk is decreased with projects have low variability
and are negatively correlated (the diversification effect).
Answers (B), (C), and (D) are incorrect because overall risk is decreased when projects have low variability and are
negatively correlated.

11

. Answer (D) is correct. An asset is liquid if it can be converted to cash on short notice. Liquidity (marketability) risk is
the risk that assets cannot be sold at a reasonable price on short notice. If an asset is not liquid, investors will require a
higher return than for a liquid asset. The difference is the liquidity premium.
Answer (A) is incorrect because default risk is the risk that a borrower will not pay the interest or principal on a loan.
Answer (B) is incorrect because interest-rate risk is the risk to which investors are exposed because of changing interest
rates. Answer (C) is incorrect because purchasing-power risk is the risk that inflation will reduce the purchasing power of
a given sum of money.

12

. Answer (D) is correct. Liquidity risk is the possibility that an asset cannot be sold on short notice for its market
value. If an asset must be sold at a high discount, it is said to have a substantial amount of liquidity risk.
Answer (A) is incorrect because interest rate risk is caused by fluctuations in the value of an asset as interest rates
change. Its components are price risk and reinvestment-rate risk. Answer (B) is incorrect because purchasing power
risk is the risk that a general rise in the price level (inflation) will reduce what can be purchased with a fixed sum of
money. Answer (C) is incorrect because financial risk is the risk borne by shareholders, in excess of basic business risk,
that arises from use of financial leverage (issuance of fixed income securities, i.e., debt and preferred stock.

13

. Answer (A) is correct. Business risk is the risk of fluctuations in earnings before interest and taxes or in operating
income when the firm uses no debt. It depends on factors such as demand variability, sales price variability, input price
variability, and the amount of operating leverage. Financial leverage affects financial risk and is not a factor affecting
business risk.
Answer (B) is incorrect because sales price variability is a factor affecting business risk. Answer (C) is incorrect because
demand variability is a factor affecting business risk. Answer (D) is incorrect because input price variability is a factor
affecting business risk.

14

. Answer (A) is correct. Business risk is the risk of fluctuations in earnings before interest and taxes or in operating
income when the firm uses no debt. It is the risk inherent in its operations that excludes financial risk, which is the risk to
the shareholders from the use of financial leverage. Business risk depends on factors such as demand variability, sales
price variability, input price variability, and amount of operating leverage.
Answer (B) is incorrect because business risk depends on such factors as amount of operating leverage. Answer (C) is
incorrect because business risk depends on such factors as demand variability. Answer (D) is incorrect because
business risk depends on such factors as fluctuations in suppliers' prices.

15

. Statements a and b are correct; therefore, statement d is the correct choice. High uncertainty about future sales
implies high business risk. The extent to which a firms costs are fixed affects business risk. Financial leverage does not
affect the firms business risk. Financial leverage shifts the business risk between classes of ownership, that is, debt and
equity.

16

REQUIRED: The set of ratios that characterizes the firm with the greatest financial risk.

DISCUSSION: (C) A firm with high risk will have a higher debt-to-equity ratio than a low risk firm, a lower interest
coverage, and a volatile return on equity. A higher debt-to-equity ratio poses a greater risk of insolvency. Debtholders
must be paid regardless of whether the firm is profitable. Low interest coverage means that the margin of safety of
earnings before interest and taxes is small. A volatile return on equity signifies that earnings are unpredictable. Lack of
predictability increases risk.
Answer (A) are incorrect because high interest coverage suggests lower risk. Answer (B) is incorrect because a low
debt-to-equity ratio indicates a low level of risk. Answer (D) is incorrect because a low debt-to-equity ratio, high interest
coverage, and a stable return on equity suggests lower risk.
17

. Answer (D) is correct. When amounts to be paid or received are denominated in a foreign currency, exchange rate
fluctuations may result in exchange gains or losses. For example, if a U.S. firm has a receivable fixed in terms of units of
a foreign currency, a decline in the value of that currency relative to the U.S. dollar results in a foreign exchange loss.
Answer (A) is incorrect because sovereign or expropriation risk is the risk that the sovereign country in which the assets
backing an investment are located will seize the assets without adequate compensation. Answer (B) is incorrect
because sovereign or expropriation risk is the risk that the sovereign country in which the assets backing an investment
are located will seize the assets without adequate compensation. Answer (C) is incorrect because the beta value in the
capital asset pricing model for a multinational firm is the systematic risk of a given multinational firm relative to that of the
market as a whole.

18

. Answer (C) is correct. Exchange-rate risk is the risk that a foreign currency transaction will be negatively exposed to
fluctuations in exchange rates. Because O & B Company sells goods to German customers and records accounts
receivable denominated in deutsche marks, O & B Company is exposed to exchange-rate risk.
Answer (A) is incorrect because liquidity risk is the possibility that an asset cannot be sold on short notice for its market
value. Answer (B) is incorrect because business risk is the risk of fluctuations in earnings before interest and taxes or in
operating income when the firm uses no debt. Answer (D) is incorrect because price risk is a component of interest-rate
risk.

19

. Answer (A) is correct. Extension of credit in a foreign currency would result in receiving payment in less valuable
dollars if the foreign currency became less valuable. Thus one would not want to encourage granting trade credit in a
foreign country when the country's currency is expected to lose value.
Answer (B) is incorrect because investing monetary assets into nonmonetary assets is advantageous when the
monetary unit is going to lose value. Answer (C) is incorrect because it is advantageous to become a debtor when the
monetary unit is losing value. Answer (D) is incorrect because it is advantageous to become a debtor when the
monetary unit is losing value.

20

. Answer (D) is correct. A firm may buy or sell forward exchange contracts to cover liabilities or receivables,
respectively, denominated in a foreign currency. Any gain or loss on the foreign receivables or payables because of
changes in exchange rates is offset by the loss or gain on the foreign exchange contract.
Answer (A) is incorrect because in general, a firm should attempt to achieve a net monetary debtor position in countries
with currencies expected to depreciate. Answer (B) is incorrect because a net monetary creditor position should be
achieved in countries with strengthening currencies. Answer (C) is incorrect because a firm may seek to minimize its
exchange-rate risk by diversification. If it has transactions in both strong and weak currencies, the effects of changes in
rates may be offsetting.

21

. Answer (B) is correct. Political risk is the risk that a foreign government may act in a way that will reduce the value
of the company's investment. Political risk may be reduced by making foreign operations dependent on the domestic
parent for technology, markets, and supplies.
Answer (A) is incorrect because political risk may be reduced by entering into a joint venture with a company from the
host country rather than from a foreign country. Answer (C) is incorrect because refusing to pay higher wages and higher
taxes will only increase political risk. Answer (D) is incorrect because political risk may be reduced by financing with
local capital, rather than foreign capital.

22

. Answer (C) is correct. A feasible portfolio that offers the highest expected return for a given risk or the least risk for a
given expected return is called an efficient portfolio.
Answer (A) is incorrect because an optimal portfolio is a portfolio selected from the efficient set of portfolios because it is
tangent to the investor's highest indifference curve. Answer (B) is incorrect because a desirable portfolio is a nonsense
term. Answer (D) is incorrect because an effective portfolio is a nonsense term.

23

. Answer (D) is correct. An investor wants to maximize expected return and minimize risk when choosing a portfolio.
A feasible portfolio that offers the highest expected return for a given risk or the least risk for a given expected return is
an efficient portfolio. A portfolio that is selected from the efficient set of portfolios because it is tangent to the investor's
highest indifference curve is the optimal portfolio.
Answer (A) is incorrect because a portfolio is efficient if it offers the highest return for a given risk or the least risk for a
given return. Answer (B) is incorrect because the optimal portfolio is tangent to the investor's highest indifference curve.
Thus, it is the efficient portfolio with the highest utility. Answer (C) is incorrect because the optimal portfolio is efficient as
well as feasible.

24

. Answer (A) is correct. A common general definition is that risk is an investment with an unknown outcome but a
known probability distribution of returns (a known mean and standard deviation). An increase in the standard deviation
(variability) of returns is synonymous with an increase in the riskiness of a project. Risk is also increased when the
project's returns are positively (directly) correlated with other investments in the company's portfolio; that is, risk
increases when returns on all projects rise or fall together. Consequently, the overall risk is decreased when projects
have low variability and are negatively correlated (the diversification effect).
Answer (B) is incorrect because uncorrelated investments are more risky than negatively correlated investments.
Answer (C) is incorrect because correlated investments are very risky. Answer (D) is incorrect because correlated
investments are very risky.

25

. Answer (B) is correct. Business units may be treated as elements of an investment portfolio. A portfolio should be
efficient in balancing the risk with the rate of return on the portfolio. The expected rate of return of a portfolio is the
weighted average of the expected returns of the individual assets in the portfolio. The variability (risk) of a portfolio's
return is determined by the correlation of the returns of individual portfolio assets. To the extent the returns are not
perfectly positively correlated, variability is decreased. Thus, business units should be selected that increase returns and
diversify and reduce risk.
Answer (A) is incorrect because Michael Porter's model of competitive strategies has two variables: competitive
advantage and competitive scope. The strategy adopted depends on whether the advantage sought is based on lower
cost or product differentiation, and on whether the scope is broad or narrow. Answer (C) is incorrect because scenario
development is a qualitative forecasting method that involves preparing conceptual scenarios of future events, given
carefully defined assumptions. It entails writing multiple alternative but equally likely descriptions of future states. A
longitudinal scenario indicates how the current circumstances may develop, and a cross-sectional scenario describes
possible future states at a designated time. Answer (D) is incorrect because situational analysis is a method of
determining an organization's direction by systematically matching its strengths and weaknesses with its environmental
opportunities and threats (referred to as a SWOT analysis).

26

. Market, or systematic, or nondiversifiable, risk is present in a diversified portfolio; the unsystematic risk has been
eliminated.

27

. The risk remaining in diversified portfolios is systematic risk; thus, portfolio returns are commensurate with systematic
risk.

28

29

. Answer (A) is correct. The general principle is that risk and return are directly correlated. U.S. Treasury securities
are backed by the full faith and credit of the federal government and are therefore the least risky form of investment.
However, their return is correspondingly lower. Corporate first mortgage bonds are less risky than income bonds or stock
because they are secured by specific property. In the event of default, the bondholders can have the property sold to

A and b quantify the extra return investors demand to bear portfolio risk.

satisfy their claims. Holders of first mortgages have rights paramount to those of any other parties, such as holders of
second mortgages. Income bonds pay interest only in the event the corporation earns income. Thus, holders of income
bonds have less risk than stockholders because meeting the condition makes payment of interest mandatory. Preferred
stockholders receive dividends only if they are declared, and the directors usually have complete discretion in this
matter. Also, stockholders have claims junior to those of debtholders if the enterprise is liquidated.
Answer (B) is incorrect because the proper listing is mortgage bonds, subordinated debentures, income bonds, and
preferred stock. Debentures are unsecured debt instruments. Their holders have enforceable claims against the issuer
even if no income is earned or dividends declared. Answer (C) is incorrect because the proper listing is first mortgage
bonds, second mortgage bonds, income bonds, and common stock. The second mortgage bonds are secured, albeit
junior, claims. Answer (D) is incorrect because the proper listing is mortgage bonds, debentures, preferred stock, and
common stock. Holders of common stock cannot receive dividends unless the holders of preferred stock receive the
stipulated periodic percentage return, in addition to any averages if the preferred stock is cumulative.
30

. REQUIRED: The least risky security from the viewpoint of the investor.
DISCUSSION: (A) A mortgage bond is secured with specific fixed assets, usually real property. Thus, under the rights
enumerated in the bond indenture, creditors will be able to receive payments from liquidation of the property in case of
default. In a bankruptcy proceeding, these amounts are paid before any transfers are made to other creditors, including
those preferences. Hence, mortgage bonds are less risky than the others listed.
Answer (B) is incorrect because a debenture is long-term debt that is not secured (collateralized) by specific property.
Subordinated debentures have a claim on the debtors assets that may be satisfied only after senior debt has been paid
in full. Debentures of either kind are therefore more risky than mortgage bonds. Answer (C) is incorrect because an
income bond pays interest only if the debtor earns it. Such bonds are also more risky than secured debt. Answer (D) is
incorrect because unsecured debt is riskier than a mortgage bond.

31

. Answer (D) is correct. The standard deviation is a measure of the degree of compactness of the values in a
population. It is a measure of dispersion. The standard deviation is found by taking the square root of the quotient of the
sum of the squared deviations from the mean, divided by the number of items in the population. Corn has a mean return
of 20% and a standard deviation of 15%. Hence, it is not as risky as Must, which has a standard deviation of 9% relative
to a mean of only 10%. The coefficient of variation (standard deviation expected return) is much higher for Must (0.9
.10 = .9) than for Corn (.15 .20 = .75).
Answer (A) is incorrect because the existence of a higher return is not necessarily indicative of high risk. Answer (B) is
incorrect because the higher standard deviation must be viewed relative to the mean of the population. Answer (C) is
incorrect because Must does not have the higher standard deviation.

32

. Answer (A) is correct. The required rate of return on equity capital in the capital asset pricing model is the risk-free
rate (determined by government securities) plus the product of the market risk premium times the beta coefficient (beta
measures the firm's risk). The market risk premium is the amount above the risk-free rate that will induce investment in
the market. The beta coefficient of an individual stock is the correlation between the volatility (price variation) of the stock
market and that of the price of the individual stock.
Answer (B) is incorrect because the coefficient of variation is the standard deviation of an investment's returns divided
by the mean return. Answer (C) is incorrect because the standard deviation is a measure of the variability of an
investment's returns. Answer (D) is incorrect because the beta coefficient measures the sensitivity of the investment's
returns to market volatility.

33

. Answer (C) is correct. The CAPM measures the tradeoff between investment return and risk. The return is the riskfree return plus a risk premium. Thus, the CAPM theory analyzes portfolios in terms of their rate of return and risk, and
the effect of adding additional securities to the portfolio.
Answer (A) is incorrect because CAPM is a portfolio (not single security) theory. Answer (B) is incorrect because the
CAPM is a general theory that is not specific to any security trading strategies. Answer (D) is incorrect because the
CAPM applies to investment securities, not fixed assets.

34

. With a diversified portfolio, the only risk remaining is market, or systematic, risk. This is the only risk that influences
return according to the CAPM.

35

. Answer (C) is correct. The risk of concern to investors who supply capital to a diversified company is market risk.
Beta or market risk is the risk that cannot be eliminated by diversification. This type of risk is defined from the
perspective of an investor who views the investment as just one part of a diversified portfolio. It is the average risk of the
firm's investment projects calculated as the weighted average of project betas.
Answer (A) is incorrect because, in project risk assessment, the firm attempts to assess the level of risk for a given
project by considering, for a given project, either pure play betas or accounting betas (beta values determined from
accounting data rather than from regression of a company's stock returns on a stock market index). Neither is a
sufficient means for assessing the overall risk of a diversified firm. Answer (B) is incorrect because, in the pure play
method of project risk assessment, the firm attempts to estimate beta for a single product company in the same line of
business as the project being evaluated. Answer (D) is incorrect because, in project risk assessment, the firm attempts
to assess the level of risk for a given project by considering, for a given project, either pure play betas or accounting
betas (beta values determined from accounting data rather than from regression of a company's stock returns on a stock
market index). Neither is a sufficient means for assessing the overall risk of a diversified firm.

36

. Answer (A) is correct. There is a positive relationship between a firm's beta value and the discount rate applied to
cash flows. Thus, a decrease in beta value will reduce the discount rate.
Answer (B) is incorrect because the discount rate will be reduced, not increased. Answer (C) is incorrect because the
discount rate will be reduced. Answer (D) is incorrect because a zero discount rate incorrectly suggests that future cash
flows do not need to be discounted for evaluation purposes.

37

. REQUIRED: The riskier stock.


DISCUSSION: (E) The standard deviation is a measure of the degree of compactness of the values in a population. It
is a measure of dispersion. The standard deviation is found by taking the square root of the quotient of the sum of the
squared deviations from the mean, divided by the number of items in the population. In other words, the standard
deviation describes how far from the mean the various elements of the population are. Cornhusker has a mean return of
20% and a standard deviation of 15%. Hence, it is not as risky as Mustang, which has a standard deviation of 9%
relative to a mean of only 10%. The coefficient of variation (standard deviation expected return) is must higher for
Mustang (0.09 0.10 = 0.9) than for Cornhusker (0.15 0.20 = 0.75).
Answer (A) is incorrect because the existence of a higher return is not necessarily indicative of high risk. Answer (B) is
incorrect because is incorrect because the higher standard deviation must be viewed relative to the mean of the
population; the absolute level of the standard deviation is meaningless without a knowledge of the mean. Answer (C) is
incorrect because Mustang does not have the higher standard deviation. Answer (D) is incorrect because the absolute
level of Mustangs rate of return is not indicative of risk.

38

. Answer (D) is correct. An important measurement used in portfolio analysis is the covariance. It measures the
volatility of returns together with their correlation with the returns of other securities. For two stocks X and Y, if k is the
expected return, ki is a given outcome, and pi is its probability, the covariance of X and Y is
n

COV(XY) (k xi k x )(k yi k y )p i
i 1

Answer (A) is incorrect because the variance and standard deviation are calculated for a single investment. The
standard deviation gives an exact value for the tightness of the distribution and the riskiness of the investment. The
standard deviation () is the square root of the variance. Answer (B) is incorrect because the variance and standard
deviation are calculated for a single investment. The standard deviation gives an exact value for the tightness of the
distribution and the riskiness of the investment. The standard deviation () is the square root of the variance. Answer (C)
is incorrect because the coefficient of variation is calculated for a single investment. The coefficient of variation is useful
when the rates of return and standard deviations of two investments differ. It measures the risk per unit of return
because it divides the standard deviation by the expected return.
39

. Answer (B) is correct. The covariance measures the volatility of returns together with their correlation with the
returns of other securities. It is calculated with the following equation:

COV(XY) (k xi k x )(k yi k y )p i
i 1

The covariance of two stocks is the same regardless of which stock is compared to the other.
Answer (A) is incorrect because the covariance of stock B with stock A is the same as the covariance of stock A with
stock B. Answer (C) is incorrect because the covariance of stock B with stock A is the same as the covariance of stock A
with stock B. Answer (D) is incorrect because the covariance of stock B with stock A is the same as the covariance of
stock A with stock B.
40

. Answer (C) is correct. An indifference curve represents combinations of portfolios having equal utility to the investor.
Given that risk and returns are plotted on the horizontal and vertical axes, respectively, and that the investor is risk
averse, the curve has an increasingly positive slope. As risk increases, the additional required return per unit of
additional risk also increases. The steeper the slope of an indifference curve, the more risk averse an investor is. The
higher the curve, the greater is the investor's level of utility.
Answer (A) is incorrect because the slope is less steep if the investor is less risk averse. The increase in the required
additional return per unit of additional risk is lower. Answer (B) is incorrect because the higher the curve, the greater is
the investor's level of utility. Moreover, the slope is positive, not negative. Answer (D) is incorrect because the slope is
increasingly positive.

41

. Answer (A) is correct. The expected rate of return of a portfolio is the weighted average of the expected returns of
the individual assets in the portfolio.
Answer (B) is incorrect because the interest rate for the safest possible investment is directly used in the capital asset
pricing model. Answer (C) is incorrect because the expected rate of return on the market portfolio is the one systematic
risk factor in the model. Answer (D) is incorrect because the standard deviation of the market returns is used to compute
the beta coefficients for each security held in the investment portfolio and the beta coefficient for the market portfolio.

42

. Answer (D) is correct. The correlation coefficient (r) measures the degree to which any two variables are related. It
ranges from -1.0 to 1.0. Perfect positive correlation (1.0) means that the two variables always move together. Perfect
negative correlation (-1.0) means that the two variables always move inversely to one another. A neutral correlation, or
no correlation, is 0.0. Skewed is a nonsense concept in this context.
Answer (A) is incorrect because returns on two stocks can be correlated in any value that falls within the range of -1.0 to
1.0. Answer (B) is incorrect because returns on two stocks can be correlated in any value that falls within the range of
-1.0 to 1.0. Answer (C) is incorrect because returns on two stocks can be correlated in any value that falls within the
range of -1.0 to 1.0.

43

. Answer (D) is correct. Maturity matching, or equalizing the life of an asset and the debt instrument used to finance
that asset, is a hedging approach. The basic concept is that the company has the entire life of the asset to recover the
amount invested before having to pay the lender.
Answer (A) is incorrect because working capital management is short-term asset management. Answer (B) is incorrect
because return maximization is more aggressive than maturity matching. It entails using the lowest cost forms of
financing. Answer (C) is incorrect because financial leverage is the relationship between debt and equity financing.

44

. Answer (A) is correct. Swaps are contracts to hedge risk by exchanging cash flows. In an interest-rate swap, one
firm exchanges its fixed interest and principal payments for a series of payments based on a floating rate. If a firm has
debt with fixed charges, but its revenues fluctuate with interest rates, it may prefer to swap for cash outflows based on a
floating rate. The advantage is that revenues and the amounts of debt service will then move in the same direction, and
interest-rate risk will be reduced. A currency swap is an exchange of an obligation to pay out cash flows denominated in
one currency for an obligation to pay in another. For example, a U.S. firm with revenues in francs has to pay suppliers
and workers in dollars, not francs. To minimize exchange-rate risk, it might agree to exchange francs for dollars held by
a firm that needs francs. The exchange rate will be an average of the rates expected over the life of the agreement.
Answer (B) is incorrect because contracts to hedge risk by exchanging cash flows include interest-rate swaps and
currency swaps. Answer (C) is incorrect because contracts to hedge risk by exchanging cash flows include interest-rate
swaps and currency swaps. Answer (D) is incorrect because contracts to hedge risk by exchanging cash flows include

interest-rate swaps and currency swaps.


45

. Answer (C) is correct. Derivatives, including options and futures, are contracts between the parties who contract.
Unlike stocks and bonds, they are not claims on business assets. A futures contract is entered into as either a
speculation or a hedge. Speculation involves the assumption of risk in the hope of gaining from price movements.
Hedging is the process of using offsetting commitments to minimize or avoid the impact of adverse price movements.
Answer (A) is incorrect because hedging decreases risk by using offsetting commitments that avoid the impact of
adverse price movements. Answer (B) is incorrect because speculation involves the assumption of risk in the hope of
gaining from price movements. Answer (D) is incorrect because speculating increases risk while hedging offsets risk.

46

. Answer (B) is correct. A put option is the right to sell stock at a given price within a certain period. If the market price
falls, the put option may allow the sale of stock at a price above market, and the profit of the option holder will be the
difference between the price stated in the put option and the market price, minus the cost of the option, commissions,
and taxes. The company that issues the stock has nothing to do with put (and call) options.
Answer (A) is incorrect because a call option is the right to purchase shares at a given price within a specified period.
Answer (C) is incorrect because selling a put option could force the company to purchase additional stock if the option is
exercised. Answer (D) is incorrect because a warrant gives the holder a right to purchase stock from the issuer at a
given price (it is usually distributed along with debt).

47

. Answer (C) is correct. The goal of duration hedging is not to equate the duration of assets and the duration of
liabilities but for the following relationship to apply: (Value of assets) X (Duration of assets)(Value of liabilities) X
(Duration of liabilities)The firm is immunized against interest-rate risk when the total price change for assets equals the
total price change for liabilities.
Answer (A) is incorrect because if duration increases, the volatility of the price of a debt instrument. Answer (B) is
incorrect because the goal is to equate the total price change for assets and the total price change for liabilities. Answer
(D) is incorrect because duration is lower if the nominal rate on a debt instrument is higher.

48

. Answer (C) is correct. A forward contract is an executory contract in which the parties involved agree to the terms of
a purchase and a sale, but performance is deferred. Accordingly, a forward contract involves a commitment today to
purchase a product on a specific future date at a price determined today.
Answer (A) is incorrect because the price of a future contract is determined on the day of commitment, not some time in
the future. Answer (B) is incorrect because performance is deferred in a future contract, and the price of the product is
not necessarily its present price. The price can be any price determined on the day of commitment. Answer (D) is
incorrect because a forward contract is a firm commitment to purchase a product. It is not based on a contingency. Also,
a forward contract does not involve an exercise price (exercise price is in an option contract).

49

. Answer (B) is correct. Interest rate futures contracts involve risk-free bonds, such as U.S. Treasury bonds. When
interest rates decrease over the period of a forward contract, the value of the bonds and the forward contract increase.
Answer (A) is incorrect because the value of the forward contract will increase when interest rates decrease. Answer
(C) is incorrect because the value of the forward contract will increase when interest rates decrease. Answer (D) is
incorrect because any decline in interest rates increases the value of the bonds.

50

. Answer (D) is correct. A distinguishing feature of futures contracts is that their prices are marked to market every
day at the close of the day. Thus, the market price is posted at the close of business each day. A mark-to-market
provision minimizes a futures contract's chance of default because profits and losses on the contracts must be received
or paid each day through a clearinghouse.
Answer (A) is incorrect because both a forward contract and a futures contract are executory. Answer (B) is incorrect
because a futures contract may be speculative. Answer (C) is incorrect because a futures contract is for delivery during
a given month.

51

. Answer (B) is correct. A change in prices can be minimized or avoided by hedging. Hedging is the process of using
offsetting commitments to minimize or avoid the impact of adverse price movements. The automobile company desires
to stabilize the price of steel so that its cost to the company will not rise and cut into profits. Accordingly, the automobile

company uses the futures market to create a long hedge, which is a futures contract that is purchased to protect against
price increases.
Answer (A) is incorrect because a short hedge is a futures contract that is sold to protect against price declines. The
automobile company wishes to protect itself against price increases. Answer (C) is incorrect because the automobile
company needs to purchase futures in order to protect itself from loss, not sell futures. Selling futures protects against
price declines. Answer (D) is incorrect because it is the definition of a short hedge, which is used for avoiding price
declines. The automobile company wants to protect itself against price increases.
52

. Answer (D) is correct. In an interest rate swap, two companies exchange their debt servicing obligations. Such a
transaction arises because the companies (called the counter parties) wish to limit interest rate risk. If a company with a
variable-rate loan desires the certainty of a fixed-rate loan, it may swap with a company that prefers the floating rate.
The actual exchange of funds during the agreement is in the form of a net payment from the party owing the greater
amount for the period. Interest rate swaps are a form of off-balance-sheet financing because only the original borrowing,
not the swap agreement, appears on the balance sheet.
Answer (A) is incorrect because an interest rate swap is not a sale of a right to borrow at a preferential rate. Answer (B)
is incorrect because the parties to an interest rate swap deal only with each other. Answer (C) is incorrect because an
interest rate swap is an exchange of interest obligations, not an offsetting of interest income and interest expense.

53

. Answer (A) is correct. Swaps are contracts to hedge risk by exchanging cash flows. In an interest-rate swap, one
firm exchanges its fixed interest and principal payments for a series of payments based on a floating rate. If a firm has
debt with fixed charges, but its revenues fluctuate with interest rates, it may prefer to swap for cash outflows based on a
floating rate. The advantage is that revenues and the amounts of debt service will then move in the same direction, and
interest-rate risk will be reduced. A currency swap is an exchange of an obligation to pay out cash flows denominated in
one currency for an obligation to pay in another. For example, a U.S. firm with revenues in francs has to pay suppliers
and workers in dollars, not francs. To minimize exchange-rate risk, it might agree to exchange francs for dollars held by
a firm that needs francs. The exchange rate will be an average of the rates expected over the life of the agreement.
Answer (B) is incorrect because contracts to hedge risk by exchanging cash flows include interest-rate swaps and
currency swaps. Answer (C) is incorrect because contracts to hedge risk by exchanging cash flows include interest-rate
swaps and currency swaps. Answer (D) is incorrect because contracts to hedge risk by exchanging cash flows include
interest-rate swaps and currency swaps.

54

. Answer (A) is correct. Under sensitivity analysis, forecasts of many calculated NPVs under various assumptions are
compared to see how sensitive the NPV of a possible project is to the change of a single input variable. In this instance,
the single unit variable being changed is the price of a table. Answer (B) is incorrect because simulation analysis
requires the use of a computer to generate many examples of results based upon various assumptions.
Answer (C) is incorrect because the informal method calculates NPVs at the firm's discount rate (k) and individually
reviews the possible projects. Answer (D) is incorrect because certainty equivalent adjustments are not frequently used
since decision makers are not familiar with its concept. It forces the decision maker to specify at what point the firm is
indifferent to the choice between a certain sum of money and the expected value of a risky sum.

55

. Answer (A) is correct. Simulation analysis considers both the sensitivity of the NPV when variables are changed
and the range of values of the variables that are changed.
Answer (B) is incorrect because The Capital Asset Pricing Model is derived from the use of portfolio theory. Answer (C)
is incorrect because sensitivity analysis only considers the sensitivity of the NPV when variables are changed, not the
range of values of the variables that are changed. Answer (D) is incorrect because certainty equivalent adjustments
does not involve calculating NPVs.

56

. Answer (D) is correct. Risk analysis attempts to measure the likelihood of the variability of future returns from the
proposed investment. Risk can be incorporated into capital budgeting decisions in a number of ways, one of which is to
use a hurdle rate higher than the firm's cost of capital, that is, a risk-adjusted discount rate. This technique adjusts the
interest rate used for discounting upward as an investment becomes riskier. The expected flow from the investment
must be relatively larger, or the increased discount rate will generate a negative net present value, and the proposed
acquisition will be rejected. Accordingly, the IRR (the rate at which the NPV is zero) for a rejected investment may

exceed the cost of capital when the risk-adjusted rate is higher than the IRR. Conversely, the IRR for an accepted
investment may be less than the cost of capital when the risk-adjusted rate is less than the IRR. In this case, the
investment presumably has very little risk. Furthermore, risk-adjusted rates may also reflect the differing degrees of risk,
not only among investments, but by the same investments undertaken by different organizational subunits.
Answer (A) is incorrect because discount rates may vary with the project or with the subunit of the organization. Answer
(B) is incorrect because the company may accept some projects with IRRs less than the cost of capital, or reject some
project with IRRs greater than the cost of capital. Answer (C) is incorrect because the company may accept some
projects with IRRs less than the cost of capital, or reject some project with IRRs greater than the cost of capital.
57

. ks = 10% + (16% - 10%)1.5 = 10% + 9% = 19%.


Expected return = 21%. 21% - Risk adjustment 1% = 20%.
Risk-adjusted return = 20% > ks = 19%.

58

Calculate the beta of the firm, and use to calculate project beta:
ks = 0.16 = 0.10 + (0.05)bFirm. bFirm = 1.2.
bProject = (bFirm)1.5. (bProject is 50% greater than current bFirm)
bProject = (1.2)1.5 = 1.8.
Calculate required return on project, kProject, and compare to expected return:
Project: kProject = 0.10 + (0.05)1.8 = 0.19 = 19%. Expected return = 0.18 = 18%. Since the required return is one
percentage point greater than the expected return, the firm should not accept the new project.

59

Calculate the beta of the firm, and use to calculate project beta:
ks = 0.16 = 0.10 + (0.05)bFirm. bFirm = 1.2.
bProject = (bFirm)1.5
(bProject is 50% greater than current bFirm)
bProject = (1.2)1.5 = 1.8.
Calculate required return on project, kProject, and compare to IRR.
Project: kProject = 0.10 + (0.05)1.8 = 0.19 = 19%. IRR = 0.18 = 18%.
Since the required return is one percentage point greater than the expected IRR, the firm should not accept the new
project.

60

. Answer (C) is correct. The theory underlying the cost of capital is based primarily on the cost of long-term funds and
the acquisition of new funds. The reason is that long-term funds are used to finance long-term investments. For an
investment alternative to be viable, the return on the investment must be greater than the cost of the funds used. The
objective in short-term borrowing is different. Short-term loans are used for working capital, not to finance long-term
investments.
Answer (A) is incorrect because the concern is with the cost of new funds; the cost of old funds is a sunk cost and of no
relevance for decision-making purposes. Answer (B) is incorrect because the cost of short-term funds is not usually a
concern for investment purposes. Answer (D) is incorrect because there is less concern with the cost of old funds or
short-term funds.

61

. Answer (C) is correct. The cost of capital is the minimum, not maximum, rate of return that must be earned on new
investments so as not to dilute shareholder interest. If new investments have a rate of return less than the cost of
capital, a loss will be incurred on those investments. Thus, the cost of capital must be the minimum rate of return. See
SMA 4A, Cost of Capital.
Answer (A) is incorrect because cost of capital is used to make capital investment decisions so that each investment
returns more than the cost of capital. Answer (B) is incorrect because the cost of maintaining working capital is based on
the cost of capital. Answer (D) is incorrect because the performance of individual investments, investment managers,
and others can be related to the cost of capital.

62

. Answer (D) is correct. The U-shaped curve indicates that the cost of capital is quite high when the debt-to-equity
ratio is quite low. As debt increases, the cost of capital declines as long as the cost of debt is less than that of equity.
Eventually, the decline in the cost of capital levels off because the cost of debt ultimately rises as more debt is used.

Additional increases in debt (relative to equity) will then increase the cost of capital. The implication is that some debt is
present in the optimal capital structure because the cost of capital initially declines when debt is added. However, a point
is reached at which debt becomes excessive and the cost of capital begins to rise.
Answer (A) is incorrect because the composition of the capital structure affects the cost of capital since the components
have different costs. Answer (B) is incorrect because the cost of debt does not remain constant as financial leverage
increases. Eventually, that cost also increases. Answer (C) is incorrect because the initial decline in the U-shaped graph
indicates that the financial markets reward moderate levels of debt.
63

. Answer (A) is correct. The overall cost of capital is the rate of return on a firm's assets that exactly covers the costs
associated with the funds employed. It is the weighted average of the various debt and equity components.
Answer (B) is incorrect because the cost of capital is based on what a company pays for its capital, not the return
earned on the capital employed. Answer (C) is incorrect because the overall cost of capital is the minimum rate a firm
must earn on all investments to cover capital costs. Answer (D) is incorrect because the overall cost of capital is based
on both debt and equity components.

64

. Answer (A) is correct. An imputed cost is one that has to be estimated. It is a cost that exists but is not specifically
stated and is the result of a process designed to recognize economic reality. An imputed cost may not require a dollar
outlay formally recognized by the accounting system, but it is relevant to the decision-making process. For example, the
stated interest on a bank loan is not an imputed cost because it is specifically stated and requires a dollar outlay. But the
cost of using retained earnings as a source of capital is unstated and has to be imputed.
Answer (B) is incorrect because the cost of obsolete assets should be written off. Answer (C) is incorrect because
understated depreciation results in unstated costs. Answer (D) is incorrect because the lower-than-market return on a
loan is an imputed cost of the products.

65

. REQUIRED: The implicit cost of financing.


DISCUSSION: (B) Debt capital often appears to have a lower cost than equity because the implicit costs are not
obvious. The implicit costs are attributable to the increased risk created by the additional debt burden. Thus, as the
debt-to-equity ratio increases, the cost of both debt and equity will increase given the increased risk to both
shareholders and creditors from a higher degree of leverage. An explanation based on the marginal cost of capital and
the marginal efficiency of investment leads to the same conclusion. Lower cost capital sources are used first. Additional
projects must then be undertaken with funds from higher cost sources. Similarly, risk is increased because the most
profitable investments are made initially, leaving the less profitable investments for the future.
Answer (A) is incorrect because both debt and equity sources increase in cost as leverage increases. Answer (C) is
incorrect because equity costs decline as leverage decreases. Answer (D) is incorrect because the weighted-average
cost of capital will increase with increased leverage.

66

. Retained earnings are just another form of equity. When the company has retained earnings, they can do one of
two things--reinvest them or pay them out as dividends. If they reinvest the earnings, they need to earn a return that is at
least as high as the ks of the stock. Otherwise, investors would be happier getting the dividends and investing them in
something that will get them ks. Therefore, statement a is false. Some of the preferred stock dividends are excluded from
taxation when another company owns them. It makes no tax difference to the company that pays the dividends, since
dividends come out of after-tax dollars. Therefore, statement b is false. Interest payments are tax deductible. Therefore,
statement c is true.

67

. Answer (B) is correct. The cost of debt capital is simply the debt interest rate times (1 - the firm's tax rate). Thus, if
the tax rate is 40%, the effective cost of debt capital is 60% times the interest rate because the interest is tax deductible.
See SMA 4A, Cost of Capital.
Answer (A) is incorrect because it ignores the tax deductibility of interest payments. Answer (C) is incorrect because it
ignores the tax deductibility of interest payments. Answer (D) is incorrect because the capital asset pricing model is one
means of determining the cost of common equity.

68

. Answer (D) is correct. The after-tax cost of debt is the cost of debt times the quantity one minus the tax rate. For
example, the after-tax cost of a 10% bond is 7% [10% x (1 - 30%)] if the tax rate is 30%.

Answer (A) is incorrect because the after-tax cost of debt is the cost of debt times the quantity one minus the tax rate.
Answer (B) is incorrect because the after-tax cost of debt is the cost of debt times the quantity one minus the tax rate.
Answer (C) is incorrect because the cost of debt times the marginal tax rate equals the tax savings from issuing debt.
69

. Answer (D) is correct. Because interest is deductible for tax purposes, the actual cost of debt capital is the net effect
of the interest payment and the offsetting tax deduction. The actual cost of debt equals the interest rate times one minus
the marginal tax rate. Thus, if a firm with an 8% market rate is in a 40% tax bracket, the net cost of the debt capital is
4.8% [8% x (1 - .4)].
Answer (A) is incorrect because the tax deduction always causes the market yield rate to be higher than the cost of debt
capital. Answer (B) is incorrect because additional debt may or may not be issued more cheaply than earlier debt,
depending upon the interest rates in the market place. Answer (C) is incorrect because the cost of debt is less than the
yield rate given that bond interest is tax deductible.

70

. Answer (C) is correct. As a larger proportion of an entity's capital is provided by debt, the debt becomes riskier and
more expensive, i.e., requires a higher interest rate.
Answer (A) is incorrect because the diversity decreases, not increases, risk. Answer (B) is incorrect because $50 million
is minuscule in the debt markets. Answer (D) is incorrect because the combination alternative maintains the same
debt/equity mixture, which would not warrant a rate increase in the cost of debt or equity.

71

. Answer (C) is correct. The excess of the price over the face value is a premium. A premium is paid because the
coupon rate on the bond is greater than the market rate of interest. In other words, because the bond is paying a higher
rate than other similar bonds, its price is bid up by investors.
Answer (A) is incorrect because, if a bond sells at a premium, the market rate of interest is less than the coupon rate.
Answer (B) is incorrect because a bond sells at a discount when the price is less than the face amount. Answer (D) is
incorrect because a bond sells at a discount when the price is less than the face amount.

72

. REQUIRED: The definition of the marginal cost of debt.


DISCUSSION: (C) The marginal cost of debt is calculated as the cost of new debt times one minus the marginal tax
rate, or Kd (1 - T). This d expression equals Kd - KdT .
Answer (A) is incorrect because the marginal cost of debt financing is the interest rate on new debt minus the firm's
marginal tax rate multiplied by the interest rate. Answer (B) is incorrect because the marginal cost of debt financing is
the interest rate on new debt minus the firm's marginal tax rate multiplied by the interest rate. Moreover, the marginal or
incremental cost of debt to the firm is based on the cost of newly issued debt, not on the cost of outstanding debt.
Answer (D) is incorrect because the marginal cost of debt financing is the interest rate on new debt minus the firm's
marginal tax rate multiplied by the interest rate. Moreover, the marginal or incremental cost of debt to the firm is based
on the cost of newly issued debt, not on the cost of outstanding debt.

73

. The debt cost used to calculate a firms WACC is kd(1 - T). If kd remains constant but T increases, then the
term (1 - T) decreases and the value of the entire equation, kd(1 - T), decreases. Statement b is false; if a
companys stock price increases, and all else remains constant, then the dividend yield decreases and ks
decreases. This can be seen from the equation ks = D1/P0 + g. Statement c is false for the same reason. The cost
of issuing new common stock is ke = D1/[P0(1 - F)] + g. If P0 increases but theres no change in the flotation cost,
ke will decrease.

74

. Answer (B) is correct. Providers of equity capital are exposed to more risk than are lenders because the firm is not
obligated to pay them a return. Also, in case of liquidation, creditors are paid before equity investors. Thus, equity
financing is more expensive than debt because equity investors require a higher return to compensate for the greater
risk assumed.
Answer (A) is incorrect because the obligation to repay at a specific maturity date reduces the risk to investors and thus
the required return. Answer (C) is incorrect because the demand for equity capital is directly related to its greater cost to
the issuer. Answer (D) is incorrect because dividends are based on managerial discretion and may rarely change;
interest rates, however, fluctuate daily based upon market conditions.

75

. Statement a is correct. Most preferred stock is owned by corporations which receive a 70 percent exclusion
of dividends. Consequently, the before-tax coupons on preferred stock are often lower than the before-tax
coupons on debt, despite the fact that preferred stock is riskier than debt. All the other statements are false.

76

Answer (B) is correct. The dividend growth model is used to calculate the price of stock.

Po =

D1
R-G

If: PO = current price


D1 = next dividend
R = required rate of return
G = EPS growth rate
Assuming that D1 and G remain constant, an increase in R resulting from an increase in the nominal interest rate will
cause PO to decrease.
Answer (A) is incorrect because a higher interest rate raises the required return of investors, which results in a lower
stock price. Answer (C) is incorrect because a higher interest rate raises the required return of investors, which results
in a lower stock price. Answer (D) is incorrect because a higher interest rate raises the required return of investors,
which results in a lower stock price.
77

. REQUIRED: The item resulting in a higher market value for common shares.
DISCUSSION: (A) The dividend growth model is used to calculate the cost of equity. The simplified formula is

R=

D
+G
P

R is the required rate of return, D is the next dividend, P is the stocks price, and G is the growth rate in earnings per
share. The equation is also used to determine the stock price.

P=

D
R-G

Thus, when investors have a lower required return on equity, the denominator is smaller, which translates in to a higher
market value.
Answer (B) is incorrect because, if investors expect lower dividend growth, the market value of common shares will
decrease. Answers (C) and (D) are incorrect because the expected holding periods of investors are not related to the
market value of the common shares.
78

. Answer (B) is correct. Newly issued or external common equity is more costly than retained earnings. The company
incurs issuance costs when raising new, outside funds.
Answer (A) is incorrect because the cost of retained earnings is the rate of return stockholders require on equity capital
the firm obtains by retaining earnings. The opportunity cost of retained funds will be positive. Answer (C) is incorrect
because retained earnings will always be less costly than external equity financing. Earnings retention does not require
the payment of issuance costs. Answer (D) is incorrect because retained earnings will always be less costly than
external equity financing. Earnings retention does not require the payment of issuance costs.

79

. Statement a is correct; the other statements are false. Preferred stock dividends are not tax deductible;
therefore, the cost of preferred stock is only kp. The risk premium in the bond-yield-plus-risk premium approach
would be added to the firms cost of debt, not the risk-free rate. Preferred stock also has flotation costs.

80

. REQUIRED: The effect of a higher dividend-payout ratio.


DISCUSSION: (A) the higher the dividend-payout ratio, the sooner retained earnings are exhausted and the company
must seek external financing. Assuming the same investments are undertaken, the result is a higher marginal cost of
capital because lower-cost capital sources will be used up earlier.
Answer (B) is incorrect because the marginal cost of capital is higher. Answers (C) and (D) are incorrect because the
existence of investment opportunities is unrelated to the dividend payout.

81

. Answer (D) is correct. The marginal cost of capital is the cost of the next dollar of capital. The marginal cost
continually increases because the lower cost sources of funds are used first. The marginal cost represents a weighted
average of both debt and equity capital.
Answer (A) is incorrect because, if the cost of capital were the same as the rate of return on equity (which is usually
higher than that of debt capital), there would be no incentive to invest. Answer (B) is incorrect because the marginal cost
of capital is affected by the degree of debt in the firm's capital structure. Financial risk plays a role in the returns desired
by investors. Answer (C) is incorrect because the rate of return used for capital budgeting purposes should be at least as
high as the marginal cost of capital.

82

. Answer (B) is correct. The measure of the value of an individual stock is dependent entirely upon the stream of
future cash flows that it will produce. To determine the stock's current value, these cash flows should be discounted to
time zero (now) to obtain the stream's present value. Stocks primarily provide cash flows to investors via dividends
(including share repurchases and liquidating dividends) and capital gain (loss) at the time of sale. Once the stream of
cash flows has been discounted over a significant number of periods, it is easy to see that the dividend stream, not the
capital gain (loss) in the final period, drives the value of the stock in question. Of course, all firms do not pay a dividend.
Common financial theory, however, states that it is the intention of every firm to pay a dividend to shareholders at some
time in the future, once the firm feels it is strong enough to do so and still support future operations. After all, it is the
primary goal of a firm's management to maximize shareholder wealth. Although many factors should be considered
when purchasing a security, the primary consideration for a value-seeking investor is the future cash flow stream.
Answer (A) is incorrect because book value is a measure of the stock's worth on a company's accounting records.
Answer (C) is incorrect because the beta coefficient is a measure of how volatile the price movements of a stock are
relative to the market as a benchmark. Answer (D) is incorrect because standard deviation is a measure of risk. While
risk is a consideration for the investor, one of the fundamental concepts in finance is that there is (should be) a trade-off
between risk and return, and as long as risk is compensated for, it is not a primary consideration.

83

Answer (D) is correct. The dividend growth model is used to calculate the cost of equity. The simplified formula is
D1
R=
+G
PO
R is the required rate of return, D1 is the next dividend, PO is the stock's price, and G is the growth rate in earnings per
share. The equation is also used to determine the stock price.

Po =

D1
R-G

Answer (A) is incorrect because the model uses the growth rate in dividends and the investor's required rate of return.
Answer (B) is incorrect because the model uses the growth rate in dividends and the investor's required rate of return.
Answer (C) is incorrect because the model uses the growth rate in dividends and the investor's required rate of return.
84

. Answer (D) is correct. The Gordon (dividend) growth model requires three elements to estimate the cost of equity
capital. These are the dividends per share, the expected growth rate in dividends per share, and the market price of the
stock. Basically, the cost of equity capital can be computed as the dividend yield (dividends price) plus the growth rate.
Answer (A) is incorrect because book value per share is not a consideration in computing the cost of equity capital.
Answer (B) is incorrect because current dividends, not current earnings, per share are a requirement for the formula.
Answer (C) is incorrect because book value per share is not a consideration in computing the cost of equity capital. Also,
current dividends, not current earnings, per share are a requirement for the formula.

85

. The CAPM assumes that investors are price-takers with the same single holding period and that there are
no taxes or transaction costs.

86

87

. The SML is a measure of expected return-beta (the CML is a measure of expected return-standard deviation of
market returns). The SML provides the expected return-beta relationship for "fairly priced" securities; thus if a portfolio
manager selects securities that are underpriced and produces a portfolio with a positive alpha, this portfolio manager

The SML is a measure of expected return per unit of risk, where risk is defined as beta (systematic risk).

would receive a positive evaluation.


88

. Both the Capital Market Line and the Security Market Line depict risk/return relationships. However, the risk measure
for the CML is standard deviation and the risk measure for the SML is beta (thus c is not true; the other statements are true).

89

. In equilibrium, the marginal price of risk for a risky security must be equal to the marginal price of risk for the market. If
not, investors will buy or sell the security until they are equal.

90

. An underpriced security will have a higher expected return than the SML would predict; therefore it will plot above
the SML.

91

. Answer (A) is correct. The capital asset pricing model adds the risk-free rate to the product of the market risk
premium and the beta coefficient. The market risk premium is the amount above the risk-free rate (approximated by the
U.S. treasury bond yield) that must be paid to induce investment in the market. The beta coefficient of an individual stock
is the correlation between the price volatility of the stock market as a whole and the price volatility of the individual stock.
Answer (B) is incorrect because the price-earnings ratio is not a component of the model. Answer (C) is incorrect
because the price-earnings ratio is not a component of the model. Answer (D) is incorrect because the dividend payout
ratio is not a component of the model.

92

. The expected rate of return on any security is equal to the risk free rate plus the systematic risk of the security (beta)
times the market risk premium, E(rM - Rf).

93

. The tangency point between the capital market line and the indifference curve is the optimal portfolio for a particular
investor.

94

95

. A security with a positive alpha is one that is expected to yield an abnormal rate of return, based on the perceived risk of
the security, and thus is underpriced.

96

. Answer (B) is correct. The required rate of return on equity capital in the Capital Asset Pricing Model is the risk-free
rate (determined by government securities), plus the product of the market risk premium times the beta coefficient (beta
measures the firm's risk). The market risk premium is the amount above the risk-free rate that will induce investment in
the market. The beta coefficient of an individual stock is the correlation between the volatility (price variation) of the stock
market and that of the price of the individual stock. For example, if an individual stock goes up 15% and the market only
10%, beta is 1.5.
Answer (A) is incorrect because the coefficient of variation compares risk with expected return (standard deviation
expected return). Answer (C) is incorrect because standard deviation measures dispersion (risk) of project returns.
Answer (D) is incorrect because expected return does not describe risk.

97

. Answer (A) is correct. The word beta is derived from the regression equation for regressing the return of an
individual security (the dependent variable) to the overall market return. The beta coefficient is the slope of the
regression line. The beta for a security may also be calculated by dividing the covariance of the return on the market and
the return on the security by the variance of the return on the market.
Answer (B) is incorrect because beta equals the covariance of the returns on the market and on the security divided by
the variance of the return on the market. Answer (C) is incorrect because beta equals the covariance of the returns on
the market and on the security divided by the variance of the return on the market. Answer (D) is incorrect because beta
equals the covariance of the returns on the market and on the security divided by the variance of the return on the
market.

98

. Answer (C) is correct. The relevant risk of a security is its contribution to the portfolio's risk. It is the risk that cannot
be eliminated through diversification and is synonymous with market risk and systematic risk. The relevant risk results

A zero alpha results when the security is in equilibrium (fairly priced for the level of risk).

from factors, such as inflation, recession, and high interest rates, that affect all stocks.
Answer (A) is incorrect because company-specific risk can be eliminated through portfolio diversification. Answer (B) is
incorrect because diversifiable risk can be eliminated through portfolio diversification. Answer (D) is incorrect because
only the systematic component of total risk is relevant to security valuation.
99

Beta is a measure of how a security's return covaries with the market returns, normalized by the market variance.

100

By definition, the beta of the market portfolio is 1.

101

Once, a portfolio is diversified, the only risk remaining is systematic risk, which is measured by beta.

102

E(RS) = rf + 0(RM - rf) = rf.

103

. The market rewards systematic risk, which is measured by beta, and thus, the risk premium on a stock or portfolio varies
directly with beta.

104

. The risk premium on the market portfolio is proportional to the average degree of risk aversion of the investor population
and the risk of the market portfolio measured by its variance.

. Calculate the required return, ks, and use to calculate the WACC:
ks = 10% + 1.38(5%) = 16.9%.
WACC = 0.5(12.0%)(0.6) + 0.5(16.9%) = 12.05%.
Compare expected project return, k Project , to WACC:

105

106

But k Project = IRR = 13.0%.


Accept the project since IRRProject > WACC: 13.0% > 12.05%.
. Answer (B) is correct. To estimate the required rate of return on equity, the capital asset pricing model (CAPM) adds
the risk-free rate (determined by government securities) to the product of the beta coefficient (a measure of the firm's
risk) and the difference between the market return and the risk-free rate. Below is the basic equilibrium equation for the
CAPM.
R = RF + (RM - RF )
Thus, given a beta of 1.2, R is 11% [5% + 1.2 (10% - 5%)]. At a beta of 1.5, R is 12.5% [5% + 1.5 (10% - 5%)].
Answer (A) is incorrect because 3% equals the market return times the increase in the beta. Answer (C) is incorrect
because the company's required rate of return is affected by a change in the company's beta coefficient. Answer (D) is
incorrect because the change results in an increase of the company's required rate of return, not a decrease.

107

11% = 5% + 1.5(9% - 5%) = 11.0%; therefore, the security is fairly priced.

108

12% < 7% + 1.3(15% - 7%) = 17.40%; therefore, stock is overpriced and should be shorted.

109

A:12% - [5% + 1.2(9% - 5%)] = 2.2%.

110

111

. Answer (B) is correct. The cost of capital of a firm is the current, weighted average, after-tax cost of the firm's
various financing components. Historical costs are irrelevant.
Answer (A) is incorrect because the cost of capital is a weighted average for all sources of capital. Answer (C) is
incorrect because costs are considered after taxes. For example, the deductibility of interest must be considered.
Answer (D) is incorrect because the time value of money should be incorporated into the calculations.

Because corporations can exclude dividends for tax purposes, preferred stock often has a before-tax
market return that is less than the issuing companys before-tax cost of debt. Then, if the issuers tax rate is
zero, its component cost of preferred would be less than its after-tax cost of debt.

112

. Answer (D) is correct. A weighted average of the costs of all financing sources should be used, with the weights
determined by the usual financing proportions. The terms of any financing raised at the time of initiating a particular
project do not represent the cost of capital for the firm. When a firm achieves its optimal capital structure, the weightedaverage cost of capital is minimized.
Answer (A) is incorrect because the cost of capital is a composite, or weighted average, of all financing sources in their
usual proportions. The cost of capital should also be calculated on an after-tax basis. Answer (B) is incorrect because
the cost of capital is a composite, or weighted average, of all financing sources in their usual proportions. The cost of
capital should also be calculated on an after-tax basis. Answer (C) is incorrect because the cost of capital is a
composite, or weighted average, of all financing sources in their usual proportions. The cost of capital should also be
calculated on an after-tax basis.

113

. Answer (A) is correct. The cost of capital of a firm is the current, weighted-average, after-tax cost of the firm's
various financing components. Historical costs are irrelevant.
Answer (B) is incorrect because costs are considered after taxes. For example, the deductibility of interest must be
considered. Answer (C) is incorrect because the time value of money should be incorporated into the calculations.
Answer (D) is incorrect because the cost of capital is a weighted average for all sources of capital.

114

. If a firm paid no income taxes, its cost of debt would not be adjusted downward, hence the component cost
of debt would be higher than if T were greater than 0. With a higher component cost of debt, the WACC would
be increased. Of course, the company would have higher earnings, and its cash flows from a given project
would be high, so the higher WACC would not impede its investments, that is, its capital budget would be larger
than if it were taxed.

115

. Statements a and c are both correct; therefore, statement d is the correct choice. Statement a recites the
definition of the weighted average cost of capital. Statement c is correct because kd = kRF + LP + MRP + DRP
while ks = kRF + (kM - kRF)b. If kRF increases then the values for kd and ks will increase.

116

. Statement a is correct; the other statements are false. Statement b is incorrect; WACC is an average of debt and
equity financing. Since debt financing is cheaper and is adjusted downward for taxes, it should, when averaged with
equity, cause the WACC to be less than the cost of equity financing. Statement c is incorrect; WACC is calculated on an
after-tax basis. Statement d is incorrect; the WACC is based on marginal, not embedded, costs. Statement e is incorrect;
the cost of issuing new common stock is greater than the cost of retained earnings.

117

. WACC measures the marginal after-tax cost of capital; therefore, statement a is false. The after-tax cost of debt
financing is less than the after-tax cost of equity financing; therefore, statement b is false. The correct choice is
statement c.

118

. The preferred stock dividend is not tax deductible like the interest payment on debt. Therefore, there is no tax
benefit from preferred stock. Statement a is true. Retained earnings are equity, and equity will have a higher cost than
debt. Therefore, statement b is false. If the beta increases, investors will require a higher rate of return to hold or buy the
stock. Therefore, the cost of equity will go up, and statement c is true. Because statements a and c are true, the correct
choice is statement e.

119

. Statements a and b are true. Statement c is false. The cost of new stock is higher than the cost of retained
earnings because there are flotation costs associated with issuing new stock. Since statements a and b are true
the correct choice is statement d.

120

. Statement c is the correct choice. A tax rate increase would lead to a decrease in the after-tax cost of debt
and, consequently, the firms WACC would decrease.

121

. Statement a is correct; the other statements are false. If RPM decreases, the cost of equity will be reduced.
Answers b through e will all increase the companys WACC.

122

123

Calculate the required return, ks, and compare to the expected return, k s .
k s = 7%.
ks = kRF + (kM - kRF)b = 0.07 + (0.10 - 0.07)0.5 = 0.085 = 8.5%.
ks > k s ; 8.5% > 7.0%; reject the investment.
. Calculate the required return, ks, and use to calculate the WACC:
ks = 10% + 1.38(5%) = 16.9%.
WACC = 0.5(12.0%)(0.6) + 0.5(16.9%) = 12.05%.
Compare expected project return, kproject , to WACC:
But kproject = 13.0%.
Accept the project since k project > WACC: 13.0% > 12.05%.

124

Calculate the after-tax component cost of debt as 10%(1 - 0.3) = 7%. If the company has earnings of $100,000
and pays out 50% or $50,000 in dividends, then it will retain earnings of $50,000. The retained earnings
breakpoint is $50,000/0.4 = $125,000. Since it will require financing in excess of $125,000 to undertake any of the
alternatives, we can conclude the firm must issue new equity. Therefore, the pertinent component cost of equity
is the cost of new equity. Calculate the expected dividend per share (note this is D1) as $50,000/10,000 = $5.
Thus, the cost of new equity is $5/[($35(1 - 0.12)] + 6% = 22.23%. Jacksons WACC is 7%(0.6) + 22.23%(0.4) =
13.09%. Only the return on Project A exceeds the WACC, so only Project A will be undertaken.

125

. Answer (C) is correct. The degree of operating leverage is the percentage change in EBIT resulting from a given
percentage change in sales. Thus, it causes EBIT to be more sensitive to changes in sales.
Answer (A) is incorrect because the term credit is not applicable to leverage. Answer (B) is incorrect because financial
leverage refers to the sensitivity of EPS to EBIT. Answer (D) is incorrect because the term intrinsic is not applicable to
leverage.

126

. Answer (A) is correct. Operating leverage is based on the degree to which fixed costs are used in production. Firms
may increase fixed costs, such as by automation, to reduce variable costs. The result is a greater degree of operating
leverage (DOL), which is the percentage change in earnings before interest and taxes (net operating profit) divided by
the percentage change in unit sales. It can also be determined from the following formula, given that Q is quantity of
units sold, P is unit price, V is unit variable cost, and F is fixed cost:
Q(P - V)
Q(P - V) - F
Answer (B) is incorrect because the degree of financial leverage equals the percentage change in EPS divided by the
percentage change in net operating profit. Answer (C) is incorrect because the breakeven point is the sales volume at
which total revenue equals total costs. Answer (D) is incorrect because the degree of total (combined) leverage equals
the percentage change in EPS divided by the percentage change in sales.

127

. Answer (C) is correct. The degree of operating leverage (DOL) is a measure of the change in earnings available to
common stockholders associated with a given change in operating earnings. It is calculated, for a particular level of
sales, as:
sales revenue - variable costs
sales revenue - variable costs - fixed operating costs

Answer (A) is incorrect because DOL varies with the sales level. Answer (B) is incorrect because the degree of financial
leverage is a measure of the change in earnings available to common stockholders associated with a given change in
operating earnings. Answer (D) is incorrect because the degree of total leverage is the multiple of the degree of
operating leverage and the degree of financial leverage. Other things being equal, DOL is higher if the degree of total
leverage is higher.

128

. Answer (A) is correct. The degree of operating leverage (DOL) is equal to the percentage change in net operating
income, that is, earnings before interest and taxes (EBIT) divided by the percentage change in sales. Because interest
expenses are not included in EBIT, the DOL is not affected by a change in interest expense.
Answer (B) is incorrect because variable cost per unit affects EBIT and therefore the DOL. Answer (C) is incorrect
because quantity of units sold affects EBIT and therefore the DOL. Answer (D) is incorrect because fixed costs affect
EBIT and therefore the DOL.

129

. Answer (B) is correct. Operating leverage is a measure of the degree to which fixed costs are used in the
production process. A company with a higher percentage of fixed costs (higher operating leverage) has greater risk than
one in the same industry that relies more heavily on variable costs. The DOL equals the percentage change in net
operating income divided by the percentage change in sales. Thus, profits became more sensitive to changes in sales
volume as the DOL increases.
Answer (A) is incorrect because a firm with higher operating leverage has higher fixed costs and lower variable costs.
Answer (C) is incorrect because a firm with higher leverage will be relatively more profitable than a firm with lower
leverage when sales are high. The opposite is true when sales are low. Answer (D) is incorrect because a firm with
higher leverage is more risky. Its reliance on fixed costs is greater.

130

. Answer (B) is correct. A purchase of treasury stock involves a decrease in assets (usually cash) and a
corresponding decrease in shareholders' equity. Thus, equity is reduced and the debt-to-equity ratio and financial
leverage increase.
Answer (A) is incorrect because assets decrease when treasury stock is purchased. Answer (C) is incorrect because a
firm's interest coverage ratio is unaffected. Earnings, interest expense, and taxes will all be the same regardless of the
transaction. Answer (D) is incorrect because the purchase of treasury stock is antidilutive; the same earnings will be
spread over fewer shares. Some firms purchase treasury stock for this reason.

131

. REQUIRED: The effect of increasing the DFL.


DISCUSSION: (D) The DFL equals the percentage change in earnings available to common shareholders divided by
the percentage change in net operating income. When the DFL rises, fixed interest charges and the riskiness of the firm
rise. As a result, the variability of the returns to equity holders will increase. In other words, the standard deviation of
returns on the equity of the company rises.
Answer (A) is incorrect because an increase in the DFL increases the riskiness of the firms stock. Thus, beta rises.
Beta is a measure of the volatility of a firms stock price relative to the average stock. Answers (B) and (C) are incorrect
because systematic risk, also known as market risk, is unrelated to the DFL. Systematic risk is not specific to a
company. It is the risk associated with a companys stock that cannot be diversified because it arises from factors that
affect all stocks.

132

. Answer (C) is correct. Financial leverage is the use of borrowed money to earn money for the benefit of
shareholders. The expectation is that investment earnings will be greater than the interest paid on the borrowed funds.
Increasing debt (such as bonds) increases financial leverage.
Answer (A) is incorrect because the issuance of common stock does not increase financial leverage. No increase in
borrowed capital and fixed interest charges occurs when equity is issued. Answer (B) is incorrect because a decrease in
the dividend payout ratio would result in increased owners' equity (retained earnings), and would not increase debt
capital and financial leverage. Answer (D) is incorrect because using debt, not equity, funds to finance new investments
increases financial leverage.

133

. Answer (C) is correct. Earnings per share is less volatile in less highly leveraged firms. Lower fixed costs result in
less variable earnings when sales fluctuate.
Answer (A) is incorrect because higher leverage is associated with higher, not lower, EPS when sales exceed the
breakeven point. Answer (B) is incorrect because earnings per share is more volatile in more highly leveraged firms.
Answer (D) is incorrect because less leverage is associated with lower, not higher, EPS when sales exceed the
breakeven point.

134

. Answer (B) is correct. Financial leverage is the amount of the fixed cost of capital, principally debt, in a firm's capital
structure relative to its operating profit. Leverage creates financial risk and is directly related to the cost of capital.
Because the company is retiring bonds, the total debt is decreased. Given that the amount of debt and leverage are
directly related, a decrease in the amount of debt results in a decrease in financial leverage.
Answer (A) is incorrect because the bond retirement decreases the debt-equity ratio. Answer (C) is incorrect because
the total assets will decrease (assets will be used to retire the debt, resulting in an increased asset turnover ratio (net
sales average total assets). Answer (D) is incorrect because the interest expense avoided will increase net profit and
the return on shareholders' equity.

135

. Statement a is true; a higher EPS does not always mean that the stock price will increase. Statement b is false; a
lower WACC will mean a higher stock price. Statement c is false; EPS can increase just because shares outstanding
decline.

136

. Statement a is incorrect because BEP = EBIT/Total assets. The extent to which the firm uses debt financing does
not effect EBIT or total assets. Statement b is incorrect because firms with a high percentage of fixed costs have a high
degree of operating leverage by definition.

137

. BEP = EBIT/TA. Since they both have the same total assets and the same BEP, then EBIT must be the same for
both companies. If A has a higher debt ratio and higher interest expense than B, and they both have the same EBIT and
tax rate, then A must have a lower NI than B. Therefore, statement a is true. If A has a lower NI than B but both have the
same total assets, then As ROA (NI/TA) must be lower than Bs ROA. Therefore, statement b is true. If both companies
have the same total assets but As debt ratio is higher than Bs, then As equity must be lower (since Total assets = Total
debt + Total equity). If A has less equity, and a lower NI than B, it is not possible to judge which companys ROE (NI/EQ)
is higher.

138

. BEP = EBIT/TA. If both firms have the same BEP ratio and same total assets, then they must have the same EBIT.
Since Firm U has no debt in its capital structure, Firm U will have higher net income than Firm L because U has no
interest expense and L does. The TIE ratio is EBIT/Int. If the two companies have the same EBIT, the one with the lower
interest expense (Firm U), will have a higher TIE. Therefore, statement a is false. Firms L and U have the same EBIT,
but Firm L has a higher interest expense, so its net income will be lower than Firm U. Since ROA is equal to NI/TA, and
the two firms have the same total assets, Firm L will have a lower ROA than Firm U. Therefore, statement b is true.
Leverage will increase ROE if BEP > kd. Since BEP is 20 percent and kd is 8 percent, leverage will increase Firm Ls
ROE. Therefore, statement c is false.

139

. Statement a is false; As net income is lower than Bs due to higher interest expense, but its assets are equal to Bs,
so As ROA must be lower than Bs ROA. Statement b is false; A has the same EBIT as B, but higher interest payments
than B; therefore, As TIE is lower than Bs. Statement c is correct.

140

. Answer (B) is correct. Leverage is the amount of the fixed cost of capital, principally debt, in a firm's capital structure
relative to its operating income. It is also defined as the ratio of debt to total assets or debt to capital. Leverage, by
definition, creates financial risk, which relates directly to the question of the cost of capital. The more leverage, the
higher the financial risk, and the higher the cost of debt capital. The degree of financial leverage (DFL) is the percentage
change in earnings available to common stockholders that is associated with a given percentage change in net
operating income (earnings before interest and taxes). The more financial leverage employed, the greater the DFL, and
the riskier the firm. Whenever the return on assets is greater than the cost of debt, additional leverage is favorable. An
increase in the equity component of the capital structure, however, decreases financial leverage. Operating leverage is a
related concept based on the degree that fixed costs are used in the production process. A company with a high
percentage of fixed costs is riskier than a firm in the same industry that relies more on variable costs to produce. The
degree of operating leverage (DOL) is the percentage change in net operating income that is associated with a given
percentage change in sales. When fixed assets increase, operating leverage also increases.
Answer (A) is incorrect because financial leverage would decrease and operating leverage would increase. Answer (C)
is incorrect because financial leverage would decrease and operating leverage would increase. Answer (D) is incorrect
because financial leverage would decrease and operating leverage would increase.

141

. Answer (D) is correct. Insurance is a method of spreading losses that arise from risks to which many persons are
subject. Loss is an unanticipated diminution in economic value as opposed to normal depreciation. Risk is uncertainty
about the occurrence or the amount of loss. For example, buildings are subject to the risk of loss by fire. If the owners all
pay small fees (premiums) for insurance coverage, every participant bears part of the loss instead of a few bearing all
the loss.
Answer (A) is incorrect because risk avoidance and loss control do not transfer risk of loss. Answer (B) is incorrect
because an insurable interest is merely a potential for economic loss if an event occurs. Answer (C) is incorrect because
there must be an insurable interest, which is basically potential for loss if an event occurs. Gambling occurs when only a
bet is at risk.

142

. Answer (A) is correct. An insurance contract (a policy) must satisfy the usual requirements: offer and acceptance,
consideration, legality, and capacity of the parties. The insured must have an insurable interest in the subject matter of
the contract. Also, the subject matter generally must exist at the time of contracting. In the contract, the insurer makes a
promise to pay a stated amount for loss or injury incurred as a result of a contingent event.
Answer (B) is incorrect because the payee may be a stranger to the contract. A person who insures his/her own life
names a third party as a beneficiary. Moreover, not every insurer is an insurance company, and the contingent event
insured against must involve a risk. Answer (C) is incorrect because there is no general requirement that an insurance
contract be written. Answer (D) is incorrect because the contract may often be oral, and, if the risk is not already in
existence, the transaction is in essence a wager, not insurance.

143

. Answer (A) is correct. An insurance contract is very similar to any other contract. However, an additional
requirement is that the insured must have an insurable interest.
Answer (B) is incorrect because there is no general requirement that an insurance contract be written. Oral binders are
given every day in the insurance business. Answer (C) is incorrect because consideration is needed for the initial
formation of an insurance contract. The premium may be paid immediately, or a promise to pay is required. Answer (D)
is incorrect because the insurance company can also commit a breach by refusing to pay the proceeds of the policy
upon the occurrence of the event.

144

. Answer (A) is correct. Life insurance is usually purchased to protect against the cessation of income needed for
support of the family and to shield them from the decedent's debts.
Answer (B) is incorrect because life insurance is customarily long-term, if not for life. Answer (C) is incorrect because,
unlike other forms of insurance, life insurance does not attempt to reimburse for the actual amount of a loss since loss of
life is not measurable. Life insurance is intended to replace economic benefits lost by a person's death. Answer (D) is
incorrect because, except for term policies, life insurance differs from other kinds of coverage in providing cash value.

145

. Answer (D) is correct. Term life insurance provides protection for a specified period. Premiums are level throughout
the period. When the term ends, the insured receives no payment. Term insurance may be renewable (possibly at higher
premiums) or convertible to another form. It is the cheapest kind of life insurance.
Answer (A) is incorrect because whole life furnishes lifetime insurance protection with a cash surrender value. Answer
(B) is incorrect because an endowment policy provides life insurance protection for its duration. A cash payment is made
(the policy endows) at the end of the term. Premiums for endowment policies are higher than for whole life insurance.
Answer (C) is incorrect because a straight life or ordinary policy is whole life insurance with level premiums payable for
life.

146

. Answer (B) is correct. When one person insures the life of another, the policyholder must have an insurable interest
in the insured. That interest is found among persons who have a close family relationship or expect to suffer substantial
economic loss from the death. Business entities are thus permitted to insure key people in their organizations whose
death might have an adverse effect on profits.
Answer (A) is incorrect because a corporation has an insurable interest only in its key employees, i.e., those whose
death would cause loss to the firm. Answer (C) is incorrect because the required loss need not be so great as to cause
cessation of business. Answer (D) because one need not be an owner or an officer to be insurable as a key person.

147

. Answer (A) is correct. Fire insurance is the most standardized kind of insurance. Following the lead of New York,
almost all states have enacted a standard policy either by legislative or administrative action.
Answer (B) is incorrect because, given that fire insurance is usually written for a 1- to 3-year period, an incontestability
clause is not necessary. Such a clause bars insurer defenses after a period specified by law or the policy. Answer (C) is
incorrect because a policy may state a definite value of the insured property or simply a maximum amount of coverage
that is not conclusive as to valuation when loss occurs. A policy may thus be valued or open (unvalued). A pro rata
clause is often included (but not required) which requires the loss to be shared pro rata when there is more than one
insurer. Answer (D) is incorrect because the insurable interest merely requires the person, e.g., mortgagee, bailee, etc.,
to suffer a loss if the event insured against occurs.

148

. Answer (D) is correct. If a premium is not received by the due date, the policyholder has a grace period under state
law, usually a month or 31 days, in which to pay. After the grace period, the cash surrender value is not forfeited but can
be withdrawn or used to buy a paid-up policy.
Answer (A) is incorrect because life insurance policies often do not cover death while the insured is in the military.
Answer (B) is incorrect because life insurance policies often do not cover death as a result of a noncommercial air flight.
Answer (C) is incorrect because a lapsed policy may often be reinstated by payment of overdue premiums plus interest.

149

. Answer (B) is correct. Ordinarily, smoke, water, or other damage caused by hostile, but not friendly, fires will be
indemnified under a fire insurance policy. Hostile fires are those ignited in places where they are not meant to be. A
friendly fire is one that burns where it is intended to burn, such as a fireplace or furnace. For example, if a friendly fire is
kept within its usual container, damage caused by smoke from it will not be reimbursed.
Answer (A) is incorrect because fire insurance usually indemnifies for loss caused by hostile, but not friendly, fires.
Answer (C) is incorrect because fire insurance usually indemnifies for loss caused by hostile, but not friendly, fires.
Answer (D) is incorrect because fire insurance usually indemnifies for loss caused by hostile, but not friendly, fires.

150

. Answer (C) is correct. Arson, fraud, or another intentional act of the insured calculated to cause the damage insured
against will preclude recovery. The parties to an insurance contract have an implied duty not to bring about the very
event that is the subject matter of the policy.
Answer (A) is incorrect because agency rules do not apply; i.e., the intentional act of an agent will not be imputed to the
insured under the doctrine of respondeat superior. Arson by an agent without the actual knowledge or conspiracy of the
insured will not preclude recovery. Answer (B) is incorrect because arson is compensable unless intended by the
insured. Answer (D) is incorrect because negligence without fraud will not prevent recovery by an insured who has acted
in good faith.

151

. Answer (D) is correct. Fire insurance generally protects the insured from damage to the insured property as a result
of fire. It does not cover the insured for causing a fire on someone else's property.
Answer (A) is incorrect because malpractice insurance is a special form of liability insurance protecting professionals
from lawsuits by third parties for negligence. Answer (B) is incorrect because homeowners insurance generally contains
a liability section in the event guests are injured on the premises. Answer (C) is incorrect because a primary purpose of
automobile insurance is to protect the owner or driver from liability in the event (s)he is responsible for damage to
another person or property.

152

. Answer (C) is correct. Co-insurance is a method of sharing risk between the insurer and the insured. A co-insurance
clause typically provides that, if the policyholder insures his/her property for at least a stated percentage (usually 80%)
of its actual cash value, any loss will be paid in full up to the face amount of the policy. Thus, a co-insurance clause is
used by many property and casualty insurers to encourage policyholders to insure commercial property for an amount
that is near to its full replacement cost.
Answer (A) is incorrect because, if a policyholder does not insure the property for an amount close enough to its full
replacement cost, the policyholder must bear a proportionate part of any partial loss. Answer (B) is incorrect because the
co-insurance requirement applies only to partial losses. Total losses result in recovery of the face amount of the policy.
Answer (D) is incorrect because a co-insurance clause encourages policyholders to insure commercial property for an
amount that is near to its full replacement cost.

153

. Answer (A) is correct. Conditions precedent, called warranties, are part of the property insurance policy. Breach of a
warranty precludes recovery and results in a forfeiture. Since the law disfavors forfeitures, courts construe questions of
interpretation favorably to the insured and against the insurer that drafted the policy. However, if the parties expressly
agree that certain statements are warranties, then a court would recognize them as warranties. Since Jewelry warranted
never to display more than $10,000 of jewelry, the breach of warranty prevents recovery.
Answer (B) is incorrect because Jewelry will recover nothing; it is not entitled to $2,000. Answer (C) is incorrect because
the warranty will not be construed as a mere representation; the intention of the parties clearly was to make it a
warranty. Answer (D) is incorrect because attachment of the application to the policy is usually sufficient to make it a
part thereof. It may also be incorporated into the policy by reference to it.

You might also like